GWD - CR - 大全 附解释

更新时间:2024-01-03 18:14:01 阅读量: 教育文库 文档下载

说明:文章内容仅供预览,部分内容可能不全。下载后的文档,内容与下面显示的完全一致。下载之前请确认下面内容是否您想要的,是否完整无缺。

GWD_CR_大全

GWD6-Q32:

Proposal: Carbon dioxide and methane in the atmosphere block the escape of heat into space. So emission of these

―greenhouse‖ gases contributes to global warming. In order to reduce global warming, emission of greenhouse gases needs to be reduced. Therefore, the methane now emitted from open landfills should instead be burned to produce electricity. Objection: The burning of methane generates carbon dioxide that is released into the atmosphere. Which of the following, if true, most adequately counters the objection made to the proposal?

A. Every time a human being or other mammal exhales, there is some carbon dioxide released into the air. B. The conversion of methane to electricity would occur at a considerable distance from the landfills.

C. The methane that is used to generate electricity would generally be used as a substitute for a fuel that does not

produce any greenhouse gases when burned. D. Methane in the atmosphere is more effective in blocking the escape of heat from the Earth than is carbon dioxide. E. The amount of methane emitted from the landfills could be reduced if the materials whose decomposition produces

methane were not discarded, but recycled.

建议者说燃烧甲烷可以减小温室效应;反对者说燃烧甲烷产生二氧化碳->温室效应;题干问反对反对者; D说甲烷比二氧化碳产生的温室效应更厉害,也就是说燃烧还是能减少温室效应的。

E说如果将分解甲烷的原料循环使用,产生的甲烷会减少。这个明显不在原文的逻辑推理范围当中,和反对者的话没关系,是beyond scope。

------------------------------------------------------------------------------------------------------------ GWD6-Q33:

Crowding on Mooreville‘s subway frequently leads to delays, because it is difficult for passengers to exit from the trains. Subway ridership is projected to increase by 20 percent over the next 10 years. The Metroville Transit Authority plans to increase the number of daily train trips by only 5 percent over the same period. Officials predict that this increase is sufficient to ensure that the incidence of delays due to crowding does not increase. Which of the following, if true, provides the strongest grounds for the officials‘ prediction?

A. By changing maintenance schedules, the Transit Authority can achieve the 5 percent increase in train trips without

purchasing any new subway cars. B. The Transit Authority also plans a 5 percent increase in the number of bus trips on routes that connect to subways. C. For most commuters who use the subway system, there is no practical alternative public transportation available. D. Most of the projected increase in ridership is expected to occur in off-peak hours when trains are now sparsely used. E. The 5 percent increase in the number of train trips can be achieved without an equal increase in Transit Authority

operational costs. ------------------------------------------------------------------------------------------------------------ GWD6-Q38:

Three large companies and seven small companies currently manufacture a product with potential military applications. If the government regulates the industry, it will institute a single set of manufacturing specifications to which all ten companies will have to adhere. In this case, therefore, since none of the seven small companies can afford to convert their production lines to a new set of manufacturing specifications, only the three large companies will be able to remain in business. Which of the following is an assumption on which the author‘s argument relies?

A. None of the three large companies will go out of business if the government does not regulate the manufacture of

the product. B. It would cost more to convert the production lines of the small companies to a new set of manufacturing

specifications than it would to convert the production lines of the large companies. C. Industry lobbyists will be unable to dissuade the government from regulating the industry.

D. Assembly of the product produced according to government manufacturing specifications would be more complex

than current assembly procedures.

E. None of the seven small companies currently manufactures the product to a set of specifications that would match

those the government would institute if the industry were to be regulated. ----------------------------------------------------------------------------------------------------------- GWD6-Q40:

It is theoretically possible that bacteria developed on Mars early in its history and that some were carried to Earth by a meteorite. However, strains of bacteria from different planets would probably have substantial differences in protein

structure that would persist over time, and no two bacterial strains on Earth are different enough to have arisen on different planets. So, even if bacteria did arrive on Earth from Mars, they must have died out. The argument is most vulnerable to which of the following criticisms?

A. It fails to establish whether bacteria actually developed on Mars.

B. It fails to establish how likely it is that Martian bacteria were transported to Earth.

C. It fails to consider whether there were means other than meteorites by which Martian bacteria could have been

carried to Earth. D. It fails to consider whether all bacteria now on Earth could have arisen from transported Martian bacteria. E. It fails to consider whether there could have been strains of bacteria that originated on Earth and later died out.

逻辑虽然很难判断哪些不重要,但是有些信息一定重要。Argument的结论是So, even if bacteria

did arrive on Earth from Mars, they must have died out。论据是,no two bacterial strains on Earth are different enough to have arisen on different planets。提出另外一种可能来削弱。

-----------------------------------------------------------------------------------------------------------

GWD-TN-5 GWD7-Q5:

Exposure to certain chemicals commonly used in elementary schools as cleaners or pesticides causes allergic reactions in some children. Elementary school nurses in Renston report that the proportion of schoolchildren sent to them for treatment of allergic reactions to those chemicals has increased significantly over the past ten years. Therefore, either Renston‘s schoolchildren have been exposed to greater quantities of the chemicals, or they are more sensitive to them than schoolchildren were ten years ago.

Which of the following is an assumption on which the argument depends?

A. The number of school nurses employed by Renston‘s elementary schools has not decreased over the past ten years. B. Children who are allergic to the chemicals are no more likely than other children to have allergies to other

substances.

C. Children who have allergic reactions to the chemicals are not more likely to be sent to a school nurse now than they

were ten years ago. D. The chemicals are not commonly used as cleaners or pesticides in houses and apartment buildings in Renston. E. Children attending elementary school do not make up a larger proportion of Renston‘s population now than they did

ten years ago.

B and C are clearly different.

B is not relevant to the original argument.

C, on the other hand, is an assumption of the original argument. Try deny C, we get \ildren who have allergic reactions to the chemicals are more likely to be sent to a school nurse now than they were ten years ago.\reported increase by nurses, and the original argument that \ldren have been exposed to greater quantities of the chemicals, or they are more sensitive to them than schoolchildren were ten years ago\tion, and the best answer。

------------------------------------------------------------------------------------------------------------ GWD7-Q14:

Certain politicians in the country of Birangi argue that a 50 percent tax on new automobiles would halt the rapid increase of automobiles on Birangi‘s roads and thereby slow the deterioration of Birangi‘s air quality. Although most experts agree that such a tax would result in fewer Birangians buying new vehicles and gradually reduce the number of automobiles on Birangi‘s roads, they contend that it would have little impact on Birangi‘s air-quality problem.

Which of the following, if true in Birangi, would most strongly support the experts‘ contention about the effect of the proposed automobile tax on Birangi‘s air-quality problem?

A. Automobile emissions are the largest single source of air pollution.

B. Some of the proceeds from the new tax would go toward expanding the nonpolluting commuter rail system. C. Currently, the sales tax on new automobiles is considerably lower than 50 percent.

D. Automobiles become less fuel efficient and therefore contribute more to air pollution as they age. E. The scrapping of automobiles causes insignificant amounts of air pollution. ------------------------------------------------------------------------------------------------------------ GWD7-Q17:

Of patients over 65 years old who survived coronary bypass surgery—a procedure widely prescribed for people with heart disease—only 75 percent benefited from the surgery. Thus it appears that for one in four such patients, the doctors who advised them to undergo this surgery, with its attendant risks and expense, were more interested in an opportunity to practice their skills and in their fee than in helping the patient.

Which of the following, if true, most seriously undermines the argument?

A. Many of the patients who receive coronary bypass surgery are less than 55 years old.

B. Possible benefits of coronary bypass surgery include both relief from troubling symptoms and prolongation of life. C. Most of the patients in the survey decided to undergo coronary bypass surgery because they were advised that the

surgery would reduce their risk of future heart attacks. D. The patients over 65 years old who did not benefit from the coronary bypass surgery were as fully informed as

those who did benefit from the surgery as to the risks of the surgery prior to undergoing it. E. The patients who underwent coronary bypass surgery but who did not benefit from it were medically

indistinguishable, prior to their surgery, from the patients who did benefit. ------------------------------------------------------------------------------------------------------------

GWD7-Q18:

Although the earliest surviving Greek inscriptions written in an alphabet date from the eighth century B.C., a strong case can be made that the Greeks actually adopted alphabetic writing at least two centuries earlier. Significantly, the text of these earliest surviving Greek inscriptions sometimes runs from right to left and sometimes from left to right. Now, the Greeks learned alphabetic writing from the Phoenicians, and in the process they would surely have adopted whatever convention the Phoenicians were then using with respect to the direction of writing. Originally, Phoenician

writing ran in either direction, but by the eighth century B.C. it had been consistently written from right to left for about two centuries.

In the argument given, the two portions in boldface play which of the following roles?

A. The first is the position that the argument seeks to establish; the second reports a discovery that has been used to

support a position that the argument opposes. B. The first is the position that the argument seeks to establish; the second presents an assumption on which the

argument relies. C. The first presents evidence that is used in support of the position that the argument seeks to establish; the second

presents an assumption on which the argument relies. D. The first is an objection raised against a position that the argument opposes; the second is the position that the

argument seeks to establish. E. The first is an objection raised against a position that the argument opposes; the second is evidence that has been

used to support that position.

wonderland gg喜欢问boldface题哦,boldface题大多是把题目看懂,看准连接词就没问题了。 这题是选B吧?这段分为两部分:

1. 尽管希腊碑铭被定为BC800年,其实它应该早200年(第一个boldface)(文章所要论述的结论)。 2. G从P那里学到这种文字,they would surely 学习他们的任何写字习惯(这点并没有依据,仅仅为作者的假设)(第二个boldface),因为G所模范的P这种习惯比BC800年还要早200年,所以进而支持结论。 所以只有B符合要求。

以下是引用rosebudxiaojie在2005-9-7 17:48:00的发言:

请教如何快速做出BOLDFACE题

最简单的方法就是先判断其中一句话所起到的作用,分为两大类,要么立场要么evidence,中间的假设或者什么其他的判断不了的先不判断,缩小范围后再区分

这道就是比较简单的划线题,事实上我只看到了第一个划线句子,立即判断是position,关于是否objection扫了一眼觉得没有转折,直接从ab中进行排除,焦点在discovery和assumption,would让我直接进行判断是assumption,得出b,这个过程用了不到四十秒

Boldface题目要总结!!!

------------------------------------------------------------------------------------------------------------ GWD7-Q21:

Which of the following most logically completes the passage?

Each species of moth has an optimal body temperature for effective flight, and when air temperatures fall much below that temperature, the moths typically have to remain inactive on vegetation for extended periods, leaving them highly vulnerable to predators. In general, larger moths can fly faster than smaller ones and hence have a better chance of evading flying predators, but they also have higher optimal body temperatures, which explains why ______.

A. large moths are generally able to maneuver better in flight than smaller moths

B. large moths are proportionally much more common in warm climates than in cool climates C. small moths are more likely than large moths to be effectively camouflaged while on vegetation D. large moths typically have wings that are larger in proportion to their body size than smaller moths do

E. most predators of moths prey not only on several different species of moth but also on various species of other

insects

1.气温与体温match -> 有效飞行;2.当气温 << 体温 -> 不能有效飞行,且易被捕食; 3.大娥比小娥飞得快 -> 大娥逃生能力强; 4.大娥的最佳体温 >> 小娥的;

-> 大娥需要相应的比较高的气温环境,以保证他们的有效飞行,所以推出B选项,大娥通常在暖温气候地区。

And also in cool climates, larger moths have to stay on vegetation for more time than small moths, increasing the risk of being predated. Thus, larger moths are less common in cool climates.

------------------------------------------------------------------------------------------------------------ GWD7-Q27:

A significant number of complex repair jobs carried out by Ace Repairs have to be reworked under the company‘s warranty. The reworked jobs are invariably satisfactory. When initial repairs are inadequate, therefore, it is not because the mechanics lack competence; rather, there is clearly a level of focused concentration that complex repairs require that is elicited more reliably by rework jobs than by first-time jobs. The argument above assumes which of the following?

A. There is no systematic difference in membership between the group of mechanics who do first-time jobs and the

group of those who do rework jobs. B. There is no company that successfully competes with Ace Repairs for complex repair jobs. C. Ace Repairs‘ warranty is good on first-time jobs but does not cover rework jobs.

D. Ace Repairs does not in any way penalize mechanics who have worked on complex repair jobs that later had to be

reworked. E. There is no category of repair jobs in which Ace Repairs invariably carries out first-time jobs satisfactorily.

对于假设题,反复强调ETS定义:取非削弱。

------------------------------------------------------------------------------------------------------------

Q28: GWD-2-10

Environmentalist: The use of snowmobiles in the vast park north of Milville create sun acceptable levels of air pollution and should be banned.

Milville business spokesperson: Snowmobiling brings many out-of-towners to Milville in winter months, to the great financial benefit of many local residents. So, economics dictate that we put up with the pollution.

Environmentalist: I disagree: A great many cross-country skiers are now kept from visiting Milville by the noise and pollution that snowmobiles generate.

Environmentalist responds to the business spokesperson by doing which of the following?

A. Challenging an assumption that certain desirable outcome can derive from only one set of circumstances

B. Challenging an assumption that certain desirable outcome is outweighed by negative aspects associated with producing

that outcome C. Maintaining that the benefit that the spokesperson desires could be achieved in greater degree by a different means D. Claiming that the spokesperson is deliberately misrepresenting the environmentalist‘s position in order to be better able

to attack it E. Denying that an effect that the spokesperson presents as having benefited a certain group of people actually benefited

those people

1。A的错误在于ONLY太STRONG。―所以他们必须忍受污染‖。spokesperson的话中看不出“必须”的意思。DICTAT

E是指HAVE CONTROLLING INFLUENCE ON。DETERMINE。并没有MUST的意思。何况one set of circumstances在原文难以找到对应。

2。spokesperson说Snowmobiling带来很多人。环境学家说污染让很多人不敢来。环境学家的目的很显然是否定Snowmobiling带来很多人。从而否定给当地居民带来经济利益。即否定这个效果。所以E是合适的 lawyer

A.only one set of circumstances,无

B. certain desirable outcome is outweighed by negative aspects associated with producing that outcome这是Environmentalist的观点,不是spokeman的观点;反

C. in greater degree by a different means,无

D. misrepresenting the environmentalist’s position in order to be better able to attack it,无

E. Denying that an effect that the spokesperson presents as having benefited a certain group of people actually benefited those people:S说因为经济利益考虑,所以污染就忍忍;E说不同意,因为污染,某些人就没来;分析E的weaken方向,就是直接否定S的假设 三无一反一正确,这是CR选项的基本特征

------------------------------------------------------------------------------------------------------------ GWD7-Q30:

Printwell‘s Ink Jet Division manufactures ink-jet printers and the ink cartridges they use. Sales of its ink-jet printers have increased. Monthly revenues from those sales, however, have not increased, because competition has forced Printwell to cut the prices of its printers. Unfortunately, Printwell has been unable to bring down the cost of manufacturing a printer. Thus, despite the increase in printer sales, the Ink Jet Division must be providing the company with much smaller than it used to.

Which of the following, if true, most seriously weakens the argument?

A. Ink-jet printers in regular use frequently need new ink cartridges, and Printwell‘s printers only accept Printwell‘s ink

cartridges. B. Unlike some competing companies, Printwell sells all of its printers through retailers, and these retailers‘ costs

account for a sizable proportion of the printers‘ ultimate retail price. C. Some printer manufacturers have been forced to reduce the sale price of their ink-jet printers even more than

Printwell has. D. In the past year, no competing manufacturer of ink-jet printers has had as great an increase in unit sales of printers

as Printwell has. E. In the past year, sales of Printwell‘s ink-jet printers have increased more than sales of any other type of printer

made by Printwell. ------------------------------------------------------------------------------------------------------------ GWD7-Q37:

Escalating worldwide demand for corn has led to a sharp increase in the market price of corn, and corn prices are likely to remain high. Corn is extensive used as feed for livestock, and because profit margins are tight in the livestock business, many farmers are expected to leave the business. With fewer suppliers, meat prices will surely rise. Nonetheless, observers expect an immediate short-term decrease in meat prices.

Which of the following, if true, most helps to justify the observers‘ expectation?

A. The increase in corn prices is due more to a decline in the supply of corn than to a growth in demand for it. B. Generally, farmers who are squeezed out of the livestock business send their livestock to market much earlier than

they otherwise would. C. Some people who ate meat regularly in the past are converting to diets that include little or no meat. D. As meat prices rise, the number of livestock producers is likely to rise again.

E. Livestock producers who stay in the business will start using feed other than corn more extensively than they did in

the past. GWD7-Q38:

Journalist: Well-known businessman Arnold Bergeron has long been popular in the state, and he has often talked about running for governor, but he has never run. However, we have just learned that Bergeron has fulfilled the financial

disclosure requirement for candidacy by submitting a detailed list of his current financial holdings to the election commission. So, it is very likely that Bergeron will be a candidate for governor this year.

The answer to which of the following questions would be most useful in evaluating the journalist‘s argument?

A. Has anybody else who has fulfilled the financial disclosure requirement for the upcoming election reported greater

financial holdings than Bergeron? B. Is submitting a list of holdings the only way to fulfill the election commission‘s financial disclosure requirements? C. Did the information recently obtained by the journalists come directly from the election commission? D. Have Bergeron‘s financial holdings increased in value in recent years?

E. Had Bergeron also fulfilled the financial disclosure requirements for candidacy before any previous gubernatorial

elections? ------------------------------------------------------------------------------------------------------------ GWD7-Q41:

Magazine Publisher: Our magazine does not have a liberal bias. It is true that when a book review we had commissioned last year turned out to express distinctly conservative views, we did not publish it until we had also obtained a second review that took a strongly liberal position. Clearly, however, our actions demonstrate not a bias in favor of liberal views but rather a commitment to a balanced presentation of diverse opinions.

Determining which of the following would be most useful in evaluating the cogency of the magazine publisher‘s response?

A. Whether any other magazines in which the book was reviewed carried more than one review of the book B. Whether the magazine publishes unsolicited book reviews as well as those that it has commissioned

C. Whether in the event that a first review commissioned by the magazine takes a clearly liberal position the magazine

would make any efforts to obtain further reviews

D. Whether the book that was the subject of the two reviews was itself written from a clearly conservative or a clearly

liberal point of view E. Whether most of the readers of the magazine regularly read the book reviews that the magazine publishes

答案为C

这是EVALUATE题。对答案的两种回答能WEAKEN和SUPPORT原文结论。 如果对C回答是,则支持结论。 对C回答否,则削弱结论。

A:carried more than one review of the book,数量不是问题,问题是性质 B:是否也出版主动送上门来的书评,无关,回答了也没用啊 C:肯定回答,说明在主动寻求平衡;否定回答,说明在消极回避

D:如果原著保守,那么这家报社仍然出版两种书评;如果原著自由,那么这家报社仍然出版两种书评;有什么区别呢? E:读者读不读是另一回事;无关

------------------------------------------------------------------------------------------------------------

GWD-TN-9 GWD-12-Q2:

Scientists are discussing ways to remove excess carbon dioxide from the atmosphere by increasing the amount that is absorbed by plant life. One plan to accomplish this is to establish giant floating seaweed farms in the oceans. When the seaweed plants die, they will be disposed of by being burned for fuel.

Which of the following, if true, would indicate the most serious weakness in the plan above?

A. Some areas of ocean in the Southern Hemisphere do not contain sufficient nutrients to support large seaweed farms. B. When a seaweed plant is burned, it releases an amount of carbon dioxide comparable to the amount it has

absorbed in its lifetime. C. Even if seaweed farms prove effective, some people will be reluctant to switch to this new fuel.

D. Each year about seven billion tons of carbon dioxide are released into the atmosphere but only about five billion

tons are absorbed by plant life. E. Seaweed farms would make more money by farming seaweed to sell as nutritional supplements than by farming

seaweed to sell as fuel. ------------------------------------------------------------------------------------------------------------ GWD-12-Q11:

In general, jobs are harder to get in times of economic recession because many businesses cut back operations. However, any future recessions in Vargonia will probably not reduce the availability of teaching jobs at government-funded schools. This is because Vargonia has just introduced a legal requirement that education in government-funded schools be available, free of charge, to all Vargonian children regardless of the state of the economy, and that current student-teacher ratios not be exceeded.

Which of the following, if true, most strengthens the argument?

A. The current student-teacher ratio at Vargonia‘s government-funded schools is higher than it was during the most

recent period of economic recession. B. During recent periods when the Vargonian economy has been strong, almost 25 percent of Vargonian children have

attended privately funded schools, many of which charge substantial fees. C. Nearly 20 percent more teachers are currently employed in Vargonia‘s government-funded schools than had been

employed in those schools in the period before the last economic recession.

D. Teachers in Vargonia‘s government-funded schools are well paid relative to teachers in most privately funded

schools in Vargonia, many of which rely heavily on part-time teachers. E. During the last economic recession in Vargonia, the government permanently closed a number of the schools that it

had funded.

One thing I might not make clear is that this is a strengthen question. So a close correlation is not necessary. Only a correlation by common snese is necessary. But I still could not see the relationship by common sense in A and C.

C说的是现在(并不是这次的RECESSION)的老师就业比上次RECESSION前的就业多20%。对原文结论不起任何作用。因为你不知上次RECESSION老师就业情况。B说是最近经济好时25%学生去读私立学校,付高昂的学费,这样将来RECESSION时,就有可能这些学生中的一部分因付不起高昂的学费而去读公立学校,使学生和老师比例更大,对老师的需求也大。从而加强结论:将来RECESSION是公立学校老师不会失业。

----------------------------------------------------------------------------------------------------------- Q12: GWD-13-27

Which if the following, if true, most logically completes the argument?

Aroca County‘s public schools are supported primarily by taxes on property. The county plans to eliminate the property tax and support schools with a new three percent sales tax on all retail items sold in the county. Three percent of current retail sales is less than the amount collected through property taxes, but implementation of the plan would not necessarily reduce the amount of money going to Aroca County public schools, because ______.

A. many Aroca County residents have already left the county because of its high property taxes B. a shopping mall likely to draw shoppers from neighboring counties is about to open in Aroca County

C. at least some Aroca County parents are likely to use the money they will save on property taxes to send their

children to private schools not funded by the county D. a significant proportion of parents of Aroca County public school students do not own their homes and consequently

do not pay property taxes E. retailers in Aroca County are not likely to absorb the sales tax by reducing the pretax price of their goods ------------------------------------------------------------------------------------------------------------ GWD-12-Q15:

FastMart, a convenience store chain, is planning to add pancake syrup to the items it sells. FastMart stores do not have shelf space to stock more than one variety of syrup. Surveys of FastMart customers indicate that one-fourth of them prefer low-calorie syrup, while three-fourths prefer regular syrup. Since FastMart‘s highest priority is to maximize sales, the obvious strategy for it is to stock regular syrup.

Which of the following, if true, most seriously weakens the argument?

A. People buying pancake syrup at convenience stores, unlike those buying it at supermarkets, generally buy it only a

few times. B. People who prefer low-calorie syrup generally use about the same amount of syrup on their pancakes as those who

prefer regular syrup. C. Regular syrup does not sell for a higher price per unit than low-calorie syrup.

D. In general, customers who prefer regular syrup will buy low-calorie syrup if regular is unavailable, but those who

prefer low-calorie will not buy regular syrup. E. Sales of syrup are not expected to account for a large proportion of total dollar sales at the average FastMart store.

因为regular syrup=3/4 customer low calorie syrup= 100% customer

agree,因为如果放low calorie syrup的话,喜欢regular syrup的人也会买

GWD-12-Q16:

11

Certain genetically modified strains of maize produce a natural insecticide that protects against maize-eating insects. The insecticide occurs throughout the plant, including its pollen. Maize pollen is dispersed by the wind and often blows onto

milkweed plants that grow near maize fields. Caterpillars of monarch butterflies feed exclusively on milkweed leaves. When, in experiments, these caterpillars were fed milkweed leaves dusted with pollen from modified maize plants, they died. Therefore, use of the modified maize inadvertently imperils monarch butterflies. Which of the following, if true, most seriously weakens the argument?

A. Per unit of volume, the natural insecticide produced by the genetically modified maize plants is less toxic to insects

than are many commercial insecticides commonly used on maize plants. B. Standard weed-control practices that have been used by farmers for decades have largely eliminated milkweed

plants from certain areas where monarch-butterfly caterpillars were once common. C. The experiments showed that the caterpillars were not harmed by contact with the pollen from the genetically

modified plants unless they ingested it. D. The maize-eating insects that the natural insecticide protects against do not feed on the pollen of the maize plant. E. Airborne maize pollen tends to collect on the middle leaves of milkweed plants and monarch caterpillars feed only

on the plant‘s tender upper leaves. ----------------------------------------------------------------------------------------------------------- Q21: GWD-13-31

By competing with rodents for seeds, black ants help control rodent populations that pose a public health risk. However, a very aggressive species of blank ant, the Loma ant, which has recently invaded a certain region, has a venomous sting that is often fatal to humans. Therefore, the planned introduction into that region of ant flies, which prey on Loma ants, would benefit public health.

Which of the following, if true, most strengthens the argument?

A. Ant flies do not attack black ants other than Loma ants.

B. Loma ants are less effective than many bird species in competing with rodents for seeds.

C. Certain other species of black ants are more effective than Loma ants in competing with rodents for seeds. D. The sting of Loma ants can also be fatal to rodents.

E. The use of pesticides to control Loma ants could have harmful effects on the environment. GWD-12-Q22:

Denoma, a major consumer-electronics maker, had a sizeable decline in sales revenue for its most recent fiscal year. This result appears surprising, because electronics retailers report that although their overall sales were considerably lower than in the previous year, their sales revenue from Denoma models actually grew, largely thanks to some innovative and popular models that Denoma introduced.

Which of the following, if true, does most to explain the apparently surprising result?

A. Because of the need to educate the public about its new models‘ capabilities, Denoma‘s advertising spending was

higher than normal over the period. B. For the period at issue, Denoma‘s major competitors reported declines in revenue that were, in percentage terms,

greater than Denoma‘s. C. A significant proportion of Denoma‘s revenue comes from making components for other consumer-electronics

manufacturers. D. Unlike some of its major competitors, Denoma has no lines of business outside consumer electronics to provide

revenue when retail sales of consumer electronics are weak. E. During the period, consumer-electronics retailers sold remaining units of Denoma‘s superseded models at prices that

were deeply discounted from those models‘ original prices. ------------------------------------------------------------------------------------------------------------ GWD-12-Q29:

The Earth‘s rivers constantly carry dissolved salts into its oceans. Clearly, therefore, by taking the resulting increase in salt levels in the oceans over the past hundred years and then determining how many centuries of such increases it would have

12

taken the oceans to reach current salt levels from a hypothetical initial salt-free state, the maximum age of the Earth‘s oceans can be accurately estimated.

Which of the following is an assumption on which the argument depends?

A. The quantities of dissolved salts deposited by rivers in the Earth‘s oceans have not been unusually large during the

past hundred years. B. At any given time, all the Earth‘s rivers have about the same salt levels. C. There are salts that leach into the Earth‘s oceans directly from the ocean floor.

D. There is no method superior to that based on salt levels for estimating the maximum age of the Earth‘s oceans. E. None of the salts carried into the Earth‘s oceans by rivers are used up by biological activity in the oceans.

VOTE FOR E

D。对D取非,有些年的盐异常高,则目前的盐度被高估,比如计算出是10M年,实际为8M年。海洋最大年龄为10M年没错。不能否定结论。

E。对E取非,有些盐被消耗,则目前的盐度被低估,比如计算值为10M,实际为12M,海洋最大年龄为10M是错的。否定结论。故为答案。

by taking the resulting increase in salt levels in the oceans over the past hundred years and then determining how many centuries of such increases it would have taken the oceans to reach current salt levels from a hypothetical initial salt-free state

管你盐是怎么来的,我只计算海水里的盐增加量就足够了(就是已经包括河水加盐减去生物耗盐的可能,所以E错)。但如果过去几百年河水盐unusually多了,海盐增加量当然比以前unusually高,把这个几百年里unusually高的增加量不恰当的推广到整个历史时期里,算出来的时间当然比真实时间的短。

----------------------------------------------------------------------------------------------------------- GWD-12-Q33:

Several of a certain bank’s top executives have recently been purchasing shares in their own bank. This

activity has occasioned some surprise, since it is widely believed that the bank, carrying a large number of bad loans, is on the brink of collapse. Since the executives are well placed to know their bank‘s true condition, it might seem that their share purchases show that the danger of collapse is exaggerated. However, the available information about the bank‘s condition is from reliable and informed sources, and corporate executives do sometimes buy shares in their own company in a calculated attempt to calm worries about their company‘s condition. On balance, therefore, it is likely that the executives of the bank are following this example.

In the argument given, the two boldfaced portions play which of the following roles?

A. The first describes the circumstance the explanation of which is the issue that the argument addresses; the second

states the main conclusion of the argument. B. The first describes the circumstance the explanation of which is the issue the argument addresses; the second

states a conclusion that is drawn in order to support the main conclusion of the argument. C. The first provides evidence to defend the position that the argument seeks to establish against opposing positions;

the second states the main conclusion of the argument. D. The first provides evidence to support the position that the argument seeks to establish; the second states a

conclusion that is drawn in order to support the argument‘s main conclusion. E. Each provides evidence to support the position that the argument seeks to establish.

GWD-12-Q38:

Outbreaks of Rift Valley fever occur irregularly in East Africa, several years apart. When outbreaks do occur, they kill

thousands of cattle. A livestock vaccine against the disease exists but is rarely used. It is too expensive for farmers to use routinely, and since it is not effective until a month after vaccination, administering it after an outbreak begins helps very little. Nevertheless, experts predict that use of the vaccine will increase significantly within the next few years.

13

Which of the following, if true, provides the strongest justification for the experts‘ prediction?

A. Rift Valley fever is spread by mosquitoes, but each outbreak is so widespread that it is impractical to control it by

using insecticides. B. When an outbreak of Rift Valley fever occurs, unaffected countries often refuse to import livestock from the

countries affected by the outbreak. C. It would take less than a month for producers of the vaccine to adjust their production operations to cope with a

large increase in demand. D. Many cattle farmers in East Africa are nomadic or live in remote villages, and such farmers, who have little access to

modern veterinary medicine, are particularly hard hit by outbreaks of Rift Valley fever. E. Recently published research has shown that certain identifiable climatic conditions are almost invariably followed,

within two to five months, by an outbreak of Rift Valley fever.

以下是引用kingsoft在2004-6-13 14:03:00的发言:

选E

原文推论:1)疫苗昂贵,2)疫苗使用一个月以后才生效(发病后使用效用不大)——》疫苗不常用 专家却说:以后这个疫苗会广泛应用,问支持专家

E说那种病之前的2-5个月的天气条件几乎不变,就是说在这种病来之前的2-5月就可知道这种病即将爆发,然后就能用疫苗 其他选项无关

要support专家的观点,有两种途径:一是对条件1的否定,即价格不贵,二是对条件2的否定,即一个月前就可以避免其作用要在一个月后才能发挥的缺点,本题E答案是对2的取非加强。

------------------------------------------------------------------------------------------------------------ GWD-12-Q40:

Which of the following most logically completes the argument?

Researchers recently asked dozens of shoppers, chosen at random coming out of a FoodBasket supermarket, what they had purchased. The prices of the very same items at the nearest ShopperKing supermarket were totaled and compared with the FoodBasket total. The ShopperKing totals averaged five percent higher than the FoodBasket totals. Nevertheless, this result does not necessarily show that shoppers at ShopperKing would save money overall by shopping at FoodBasket instead, since ______.

A. shoppers who shop regularly at a given supermarket generally choose that store for the low prices offered on the

items that they purchase most often B. for shoppers with more than 20 items, the ShopperKing totals averaged more than five percent higher than the

FoodBasket totals C. many shoppers consider factors other than price in choosing the supermarket at which they shop most regularly D. there is little variation from month to month in the overall quantity of purchases made at supermarkets by a given

shopper E. none of the people who conducted the research were employees of the FoodBasket supermarket ------------------------------------------------------------------------------------------------------------

14

GWD-TN-12 T-4-Q2

Twenty years ago, Balzania put in place regulations requiring operators of surface mines to pay for the reclamation of

mined-out land. Since then, reclamation technology has not improved. Yet, the average reclamation cost for a surface coal mine being reclaimed today is only four dollars per ton of coal that the mine produced, less than half what it cost to reclaim surface mines in the years immediately after the regulations took effect.

Which of the following, if true, most helps to account for the drop in reclamation costs described?

A. Even after Balzania began requiring surface mine operators to pay reclamation costs, coal mines in Balzania

continued to be less expensive to operate than coal mines in almost any other country. B. In the twenty years since the regulations took effect, the use of coal as a fuel has declined from the level it was at

in the previous twenty years. C. Mine operators have generally ceased surface mining in the mountainous areas of Balzania because reclamation

costs per ton of coal produced are particularly high for mines in such areas. D. Even after Balzania began requiring surface mine operators to pay reclamation costs, surface mines continued to

produce coal at a lower total cost than underground mines. E. As compared to twenty years ago, a greater percentage of the coal mined in Balzania today comes from surface

mines.

原文要解释的是:今天表层煤矿回填费用的减低。(相比20年前)。C说的是现在回填费用高的山区不开采煤矿了,不需要回填拉。这样需要回填的都是费用低的,所以平均费用就降下来啦。故为答案。A和别的国家比,错。B讲的是煤的用途比例变化,无关,错。D和地下煤比,错。E 讲的是表层煤量的变化,无关。

与量有关的题目好像有共通点,有时间总结一下。

Q13 GWD-24-35(CD-10-35)

The milk of many mammals contains cannabinoids, substances that are known to stimulate certain receptors in the brain. To investigate the function of cannabinoids, researchers injected newborn mice with a chemical that is known to block

cannabinoides from reaching their receptors in the brain. The injected mice showed far less interest in feeding than normal newborn mice do. Therefore, cannabinoids probably function to stimulate the appetite. Which of the following is an assumption on which the argument depends?

A. Newborn mice do not normally ingest any substance other than their mothers‘ milk. B. Cannabinoids are the only substances in mammals‘ milk that stimulate the appetite.

C. The mothers of newborn mice do not normally make any effort to encourage their babies to feed. D. The milk of mammals would be less nutritious if it did not contain cannabinoids.

E. The chemical that blocks cannabinoids from stimulating their brain receptors does not independently inhibit the

appetite. T-4-Q14 GWD-13-38

Frobisher, a sixteenth-century English explorer, had soil samples from Canada‘s Kodlunarn Island examined for gold content. Because high gold content was reported, Elizabeth I funded two mining expeditions. Neither expedition found any gold there. Modern analysis of the island‘s soil indicates a very low gold content. Thus the methods used to determine the gold content of Frobisher‘s samples must have been inaccurate.

Which of the following is an assumption on which the argument depends?

A. The gold content of the soil on Kodlunarn Island is much lower today than it was in the sixteenth century. B. The two mining expeditions funded by Elizabeth I did not mine the same part of Kodlunarn Island.

C. The methods used to assess gold content of the soil samples provided by Frobisher were different from those

generally used in the sixteenth century. D. Frobisher did not have soil samples from any other Canadian island examined for gold content. E. Gold was not added to the soil samples collected by Frobisher before the samples were examined.

15

16

T-4-Q15

TrueSave is a mail-order company that ships electronic products from its warehouses to customers worldwide. The

company‘s shipping manager is proposing that customer orders be packed with newer, more expensive packing materials that virtually eliminate damage during shipping. The manager argues that overall costs would essentially remain unaffected, since the extra cost of the new packing materials roughly equals the current cost of replacing products returned by customers because they arrived in damaged condition.

Which of the following would it be most important to ascertain in determining whether implementing the shipping manager‘s proposal would have the argued-for effect on costs?

A. Whether the products shipped by TrueSave are more vulnerable to incurring damage during shipping than are

typical electronic products. B. Whether electronic products are damaged more frequently in transit than are most other products shipped by mail-order companies C. Whether a sizable proportion of returned items are returned because of damage already present when those items

were packed for shipping D. Whether there are cases in which customers blame themselves for product damage that, though present on arrival

of the product, is not discovered until later E. Whether TrueSave continually monitors the performance of the shipping companies it uses to ship products to its

customers T-4-Q18 天山-7-17

As a large corporation in small country, Hachnut wants its managers to have international experience, so each year it sponsors management education abroad for its management trainees. Hachnut has found, however, that the attrition rate of graduates from this program is very high, with many of them leaving Hachnut to join competing firms soon after completing the program. Hachnut does use performance during the program as criterion in deciding among candidates for management positions, but both this function and the goal of providing international experience could be

achieved in other ways. Therefore, if the attrition problem cannot be successfully addressed, Hachnut should discontinue the sponsorship program.

In the argument given, the two boldfaced portions play which of the following roles?

A. The first describes a practice that the argument seeks to justify; the second states a judgment that is used in

support of a justification for that practice. B. The first describes a practice that the argument seeks to explain; the second presents part of the argument‘s

explanation of that practice. C. The first introduces a practice that the argument seeks to evaluate; the second provides grounds for holding that

the practice cannot achieve its objective. D. The first introduces a policy that the argument seeks to evaluate; the second provides grounds for holding that the

policy is not needed. E. The first introduces a consideration supporting a policy that the argument seeks to evaluate; the second provides

evidence for concluding that the policy should be abandoned.

D更好,前面第一句话很难区分,说是practice或者policy都算对,从整个来看policy更好,因为是H锻炼他的经理所发起的一项策略。第二句关键。文章的结论:if the attrition problem cannot be successfully addressed, Hachnut should

discontinue the sponsorship program。是个条件句。说明可以通过其它的方法提供经理们国际经验和那个策略的另外一个目的。而且由于这个策略的一项缺点。所以说明这个策略是不需要的。C中不能达成目标不对,可以打成目标,但是有缺点。

T-4-Q19

Aroca City currently funds its public schools through taxes on property. In place of this system, the city plans to introduce a sales tax of three percent on all retail sales in the city. Critics protest that three percent of current retail sales falls short of the amount raised for schools by property taxes. The critics are correct on this point. Nevertheless,

implementing the plan will probably not reduce the money going to Aroca‘s schools. Several large retailers have selected Aroca City as the site for huge new stores, and these are certain to draw large numbers of shoppers from neighboring

17

municipalities, where sales are taxed at rates of six percent and more. In consequence, retail sales in Aroca City are bound to increase substantially.

In the argument given, the two potions in boldface play which of the following roles?

A. The first is an objection that has been raised against a certain plan; the second is a prediction that, if accurate,

undermines the force of that objection. B. The first is a criticism, endorsed by the argument, of a funding plan; the second is a point the argument makes in

favor of adopting a alternative plan. C. The first is a criticism, endorsed by the argument, of a funding plan; the second is the main reason cited by the

argument for its endorsement of the criticism. D. The first is a claim that the argument seeks to refute; the second is the main point used by the argument to show

that the claim is false. E. The first is a claim that the argument accepts with certain reservations; the second presents that claim in a

rewarding that is not subject to those reservations. T-4-Q20 天山-7-22

The violent crime rate (number of violent crimes per 1,000 residents) in Meadowbrook is 60 percent higher now than it was four years ago. The corresponding increase for Parkdale is only 10 percent. These figures support the conclusion that residents of Meadowbrook are more likely to become victims of violent crime than are residents of Parkdale. The argument above is flawed because it fails to take into account

A. Changes in the population density of both Meadowbrook and Parkdale over the past four years.

B. How the rate of population growth in Meadowbrook over the past four years compares to the corresponding rate for

Parkdale C. The ratio of violent to nonviolent crimes committed during the past four years in Meadowbrook and Parkdale D. The violent crime rates in Meadowbrook and Parkdale four years ago

E. How Meadowbrooks‘ expenditures for crime prevention over the past four years compare to Parkdale‘s expenditures.

答案为D无疑。

原文结论的意思实际为:现在M和P哪个犯罪率高。

在得出这个结论时原文提出的证据是和四年前比的数据,即以四年前的犯罪率作为基础,所以要比较现在的犯罪率,必须比较四年前的犯罪率。如果四年前M的犯罪率比P低很多很低,那末虽然这四年M的犯罪率增长达,但现在的犯罪率也会比P低。原文没考虑这种可能性。即D

B的意思为在过去四年里,M和P的人口增长率相比情况。因为犯罪率本身已算进了人口增长,所以不必再考虑人口增长情况。B是无关项。

这种题目好像也是通病,不能仅仅通过增长率来下结论,说增长之后的数量对比,也要考虑到基数的比较。注意这类题目。!!!

T-4-Q21

The OLEX Petroleum Company has recently determined that it could cut its refining costs by closing its Grenville refinery and consolidating all refining at its Tasberg refinery. Closing the Grenville refinery, however, would mean the immediate loss of about 1,200 jobs in the Grenville area. Eventually the lives of more than 10,000 people would be seriously disrupted. Therefore, OLEX‘s decision, announced yesterday, to keep Grenville open shows that at OLEX social concerns sometimes outweigh the desire for higher profits.

Which of the following, if true, most seriously undermines the argument given?

18

A. The Grenville refinery, although it operates at a higher cost than the Tasberg refinery, has nevertheless been

moderately profitable for many years. B. Even though OLEX could consolidate all its refining at the Tasberg plant, doing so at the Grenville plant would not

be feasible. C. The Tasberg refinery is more favorably situated than the Grenville refinery with respect to the major supply routes

for raw petroleum. D. If the Grenville refinery were ever closed and operations at the Tasberg refinery expanded, job openings at Tasberg

would to the extent possible be filled with people formerly employed at Grenville. E. Closure of the Grenville refinery would mean compliance, at enormous cost, with demanding local codes regulating

the cleanup of abandoned industrial sites.

原文说为了cut the cost,决定要去关掉G,然而由于G的关闭会引起很多社会问题。 原文就得出结论,G继续开,是考虑到社会问题

E-关掉后,要清理遗留物的cost非常高,所以继续开,说明G继续开还是考虑到成本而不是社会问题

A-profit的多少,与原文的cost cut无关,原文并没有说 G是不赚钱的,只是说成本太高了,基于降成本的考虑,刚开始决定关掉G B-排除他因,consolidate不行,仍继续开,说明是考虑社会问题; C-排除他因,没有T方便,仍继续开,说明是考虑社会问题 D-无关,问的是G的情况,与T没啥关系啊.

T-4-Q28

In Berinia, the age at which people could begin to drink alcohol legally used to be 18. In 1990, in an attempt to reduce alcohol consumption and thereby to reduce alcohol-related traffic deaths among Berinians under 21, the legal drinking age was raised to 21. Alcohol-related traffic deaths among people under 21 have decreased significantly since 1990. Nevertheless, surveys show that people in that age-group drink just as much alcohol as they did before 1990. Which of the following, if true of Berinia, most help to resolve the apparent discrepancy?

A. For the population as a whole, annual alcohol consumption is no lower now than it was in 1990

B. Alcohol consumption away from home, for example in bars and restaurants, is much lower among people under 21

than it was in 1990 C. The proportion of people under 21 who own a car is higher now than it was in 1990

D. Alcohol consumption is lower among people under 21 than among adults in most other age-groups. E. Alcohol-related traffic deaths among people over 21 have increased slightly since 1990.

原文说,新措施实施后,死的人少了很多。说明新措施是有效的;(这里文中有一个潜在推理或者假设:新措施有效是通过降低了21岁以下人的喝酒量来实现的。)最后一句说实际上21岁以下的人喝的酒并没有减少。---- 与这个潜在推理/假设矛盾。(discrepancy). B说在酒吧和餐馆里喝酒的人少了很多。这里有一个推测和一个假设,推测,新措施后,相当一部分喝酒的人从酒吧和餐馆转移到了家里;假设,在家里喝酒不会受新措施限制。基于原文喝酒的总量没变,推测成立。这个假设算是一个COMMON SENSE,所以选B.

19

T-4-Q32

Delta products, Inc., has recently switched at least partly from older technologies using fossil fuels to new technologies powered by electricity. The question has been raised whether it can be concluded that for a given level of output, Delta‘s operation now causes less fossil fuel to be consumed than it did formerly. The answer, clearly, is yes, since the amount of fossil fuel used to generate the electricity needed to power the new technologies is less than the amount needed to power the older technologies, provided that the level of output is held constant. In the argument given, the two boldface portions play which of the following roles?

A. The first identifies the context of the conclusion of the argument; the second provides support for that conclusion. B. The first provides support for the conclusion of the argument; the second identifies the content of that conclusion. C. The first states the position that the argument opposes; the second states the conclusion of the argument. D. Each provides evidence that calls the conclusion of the argument into question. E. Each provides support for conclusion of the argument. T-4-Q34

In Kravonia, the average salary for jobs requiring a college degree has always been higher than the average salary for jobs that do not require a degree. Current enrollments in Kravonia‘s colleges indicate that over the next four years the

percentage of the Kravonian workforce with college degree will increase dramatically. Therefore, the average salary for all workers in Kravonia is likely to increase over the next four years.

Which of the following is an assumption on which the argument depends?

A. Kravonians with more than one college degree earn more, on average, than do Kravonians with only one college

degree. B. The percentage of Kravonians who attend college in order to earn higher salaries is higher now than it was several

years ago. C. The higher average salary for jobs requiring a college degree is not due largely to a scarcity among the Kravonian

workforce of people with a college degree. D. The average salary in Kravonia for jobs that do not require a college degree will not increase over the next four

years. E. Few members of the Kravonian workforce earned their degrees in other countries. T-4-Q38

In one state, all cities and most towns have antismoking ordinances, a petition entitled ―Petition for Statewide Smoking Restriction‖ is being circulated to voters by campaign workers who ask only, ―Do you want to sign a petition for statewide smoking restriction?‖ The petition advocates a state law banning smoking in most retail establishments and in government offices that are open to the public.

Which of the following circumstances would make the petition as circulated misleading to voters who understand the proposal as extending the local ordinances statewide?

A. Health costs associated with smoking cause health insurance premiums to rise for everyone and so affect

nonsmokers.

B. In rural areas of the state, there are relatively few retail establishments and government offices that are open to the

public. C. The state law would supersede the local antismoking ordinances, which contain stronger bans than the state law

does. D. There is considerable sentiment among voters in most areas of the state for restriction of smoking.

E. The state law would not affect existing local ordinances banning smoking in places where the fire authorities have

determined that smoking would constitute a fire hazard.

答案应该为B。

选民的误解在于:他们认为该建议是将禁烟扩大到整个state。而根据原文,他们的建议只是in most retail establishments and

in government offices that are open to the public禁烟。所以如果如B所说在rural areas ,这些地方很

20

T-9-Q18. The growing popularity of computer-based activities was widely expected to result in a decline in television viewing, since it had been assumed that people lack sufficient free time to maintain current television-viewing levels while spending increasing amounts of free time on the computer. That assumption, however, is evidently false: in a recent mail survey concerning media use, a very large majority of respondents who report increasing time spent per week using computer report no change in time spent watching television.

Which of the following would be most useful to determine in order to evaluate the argument?

A. Whether a large majority of the survey respondents reported watching television regularly

B. Whether the amount of time spent watching television is declining among people who report that they rarely or

never use computers C. Whether the type of television programs a person watches tends to changes as the amount of time spent per week

using computer increases D. Whether a large majority of the computer owners in the survey reported spending increasing amounts of time per

week using computers E. Whether the survey respondents‘ reports of time spent using computers included time spent using computers at

work.

这题是典型的偷换概念的题。

the question is about the allocation of the people's free time spend between the TV and Computer, but the survey is the increasing time spent per week using computers ,

time spent per week using computers ==free time + work time

26

T-9-Q19. TS-7-11 In 1992 outlaw fishing boats began illegally harvesting lobsters from the territorial waters of the country of Belukia. Soon after, the annual tonnage of lobster legally harvested in Belukian waters began declining; in 1996, despite there being no reduction in the level of legal lobster fishing activity, the local catch was 9,000 tons below pre-1992 levels. It is therefore highly likely that the outlaw fishing boats harvested about 9,000 tons of lobster illegally that year. Which of the following is an assumption on which the argument depends?

A. The illegal lobster harvesting was not so extensive that the population of catchable lobsters in Belukia‘s territorial

waters had sharply declined by 1996 B. The average annual lobster catch, in tons, of an outlaw fishing boat is has increased steadily since 1992. C. Outlaw fishing boats do not, as a group, harvest more lobsters than do licensed lobster-fishing boats. D. The annual legal lobster harvest in Belukia in 1996 was not significantly less than 9,000 tons.

E. A significant proportion of Belukia‘s operators of licensed lobster-fishing boats were out of business between 1992

and 1996

题目里好象没有说1992年以来非法捕捞平均量在STEADILY增加吧,这个STEADILY是很有问题的啊。

A的意思是,非法捕捞并没有多到让龙虾数量急剧减少的程度。这样就正好说明了题意,因为龙虾总数量不变,合法捕捞减少的龙虾数量就正好被非法捕捞的人给弄走了。 文章说:1996年比1992年少了9000吨

前提是,1996年的量和1992年的量是一样的,这样的话,才会有9000吨的差距

A:就是在说1996年的龙虾量和1992年的龙虾量是差不多的,这样的话,比较才有了基础

T-9-Q25. Scientists typically do their most creative work before the age of forty. It is commonly thought that this happens because aging by itself brings about a loss of creative capacity. However, a study has found that almost all scientists who produce highly creative work beyond the age of forty entered their fields late and less than a dozen years before their creative breakthroughs. Since creative breakthroughs by scientists under forty also generally occur within a dozen years of the scientists’ entry into the field, the study‘s finding strongly suggests that the real reason why scientists over forty rarely produce highly creative work is not due to age but rather because most have spent too long in their fields.

In the argument given, the two portions in boldface play which of the following roles? A. The first is the position that the argument as a whole depends; the second is evidence that is advanced as part of

that evidence. B. The first and second are both claims that have been advanced in support of a position that the argument as a whole

opposes. C. The first is an explanation that the argument challenges; the second provides evidence in support of a competing

explanation that the argument defends. D. The first is an explanation that the argument challenges; the second is evidence that has been used against an

alternative explanation that the argument defends. E. The first is an explanation that the argument defends; the second is evidence that has been used to challenge that

explanation.

T-9-Q28. 天山-7-13 Lyme disease is caused by a bacterium transmitted to humans by deer ticks. Generally deer ticks pick up the bacterium while in the larval stage from feeding on infected white-footed mice. However, certain other species on which the larvae feed do

27

not harbor the bacterium. Therefore, if the population of these other species were increased, the number of ticks acquiring the bacterium and hence the number of people contracting Lyme disease would likely decline. Which of the following, if true, most strengthens the argument?

A. Ticks do not suffer any adverse consequences from carrying the bacterium that causes Lyme disease in humans. B. There are no known cases of a human‘s contracting Lyme disease through contact with white-footed mice. C. A deer tick feeds only once while in the larval stage.

D. A single host animal can be the source of bacterium for many tick larvae.

E. None of the other species on which deer tick larvae feed harbor other bacteria that ticks transmit to humans

其实该题单纯从有关无关就可以找到答案C

原文推理是:1。T传细菌给人类,人类感染L病。2。T在幼虫是吃M就带该细菌。3。幼虫吃其它东西不带该细菌。所以(结论),其他东西多拉,携带该细菌的T就少,人类感染L病的机会也少。

A。无关。T携带该细菌是否有反作用和结论无关

B。无关。主要有两个因素决定和结论无关。1是原文讨论的是T传细菌给人类导致人类得L病,人类直接接触M 是否得L病与原文无关。2是知不知道有没有这样的CASE并不代表实际有没有,所以对原文不起作用。

C。支持。T 在幼虫期只是进吃一次。既然只吃一次,这次不是吃M就是吃其他东西,当其他东西变多时,吃M的机率就变少,所以T感染该细菌就少。支持结论 DE都无关。

T-9-Q30. The percentage of households with an annual income of more than $40,000 is higher in Merton county than in any other county. However, the percentage of households with an annual income of $60,000 or more is highest in Sommer county. If the statements above are true, which of the following can properly be concluded on the basis of them?

A. No households in Merton county has an annul income of $60,000 or more

B. Some households in Merton county have an annual income between $40,000 and $60,000

C. The number of households with an annual income of more than $40,000 is greater in Merton than in Sommer

county. D. Average annual household income is higher in Sommer than in Merton county.

E. The percentage of households with an annual income of $80,000 is higher in Sommer than in Merton county.

这题相当于数学里头的DS:)

两个条件:1、M county年收入在40000刀以上的家庭比例高于其他county(也就是最高); 2、S county 年收入在60000刀以上的家庭比例最高 两个county斗富,看能推出啥结果。

28

A M里头没有收入高于60000的——显然不对;

B M里有人家收入在40000-60000——根据条件1能成立;

C M里收入40000以上的家庭数目比S多——条件给的是百分比,不是绝对数量;

(这个比较经典,经常通过百

分比,来错误比较实际数量)

D S的家庭平均年收入比M高——这个要看穷人还有多少,虽然他们高收入的人多(条件2),万一穷人更多,平均收入就很难说; E S里收入80000以上的比M多——不能推出。举个例子:M收入60000以上的有15%,S有16%(符合条件2),但是M这个地方有好多大款,收入80000以上的占14%,也就是说只有2%收入在60000-8000;而S这个地方贫富差距小,收入80000以上的大款才1%,这和条件不冲突,所以E是不对的。

T-9-Q32. Ecologists: the Scottish Highlands were once the site of extensive forests, but these forests have mostly disappeared and been replaced by peat bogs. The common view is that the Highland‘s deforestation was caused by human activity, especially agriculture. However, agriculture began in the Highlands less than 2,000 years ago. Peat bogs, which consist of compressed decayed vegetable matter, build up by only about one foot per 1000 years, and, throughout the Highlands, remains of trees in peat bogs are almost all at depth great than four feet. Since climate changes that occurred between 7,000 years and 4,000 years ago favored the development of peat bogs rather than the survival of forests, the deforestation was more likely the result of natural processes than of human activity. In the ecologist‘s argument, the two portions in boldfaces play which of the following roles?

A. The first is evidence that has been used in support of a position that the argument rejects; the second is a finding

that the ecologist uses to counter the evidence. B. The first is evidence that, in light of the evidence provided in the second, serves as grounds for the ecologist‘s

rejection of a certain position C. The first is a position that the ecologist rejects; the second is evidence that has been used in support of that

position. D. The first is a position that the ecologist rejects; the second provides evidence in support of that rejection. E. The first is a position for which the ecologist argues; the second provides evidence to support that position.

T-9-Q33. Which of the following, if true, most logically completes the passage?

A recent poll found that over 80 percent of the residents of Nalmed Province favored a massive expansion of the commuter rail system as a means of significantly easing congestion on the province‘s highways and were willing to help pay for the expansion through an increase in their taxes. Nevertheless, the poll results indicate that expansion of the rail system, if successfully completed, would be unlikely to achieve its goal of easing congestion, because . A. most of people in favor of expanding the rail system reported less congestion during their highway commute as the

primary benefit they would experience. B. of the less than 20 percent of residents not counted as favoring the expansion, about half claimed to have no

opinion one way or the other. C. the twice-daily periods of peak congestion caused by people commuting in cars have grown from about an hour

each to almost two and a half hours each in the past 20 years. D. expanding the commuter rail system will require the construction of dozens of miles of new railroads.

E. the proposed expansion to the commuter rail system will make it possible for some people who both live and work

at suburban locations to commute by rail.

29

选A

E-一些工作和生活都在郊区的人会使用这个铁路,的确使使用铁路的人增加,但铁路运量能否消化这种增加,原题没有给出任何信息,所以不能判断说该情况不利于铁路建造。同时,根据常识,往往高速上堵车都是发生在上下班高峰期,在国外尤其如此。因此,那些工作和住都在郊区的人一般是不会在高峰期使用铁路的,这样就和另一些需要它的人避开了

A-大部分接受调查的人都表示他们支持建造铁路的根本理由是希望减少高速路上的拥堵。这就说明,多数人还是倾向于选择高速,只是希望别人去乘铁路。所以,铁路分流高速的目的其实没有达到。 难题猜A法:)

这题有一个地方要弄清:rail和highway是两种交通方式,使用人数(也可以说是拥挤程度)是此消彼长的。题中最后说的its goal

of easing congestion,当然说的是ease highway的拥挤程度。

所以,E肯定不对。因为E说有人可以坐rail了,当然highway上的人不会多。这怎么能削弱呢?

C也不对。C说highway拥挤程度已经更严重了,这个是无关的啊。管你现在highway拥挤是不是比以前严重,都不妨碍rail对其进行缓解。

T-9-Q40. For most people, the left half of the brain controls linguistic capabilities, but some people have their language centers in the right half. When a language center of the brains is damaged, for example, by a stroke, linguistic capabilities are impaired in some way. Therefore, people who have suffered a serious stroke on the left side of the brain without suffering any such impairment must have their language centers in the right half.

Which of the following is an assumption on which the reasoning in the argument above depends?

A. No part of a person‘s brain that is damaged by a stroke never recovers

B. Impairment of linguistic capabilities does not occur in people who have not suffered any damage to any language

center of the brain. C. Strokes tend to impair linguistic capabilities more severely than does any other cause of damage to language

centers in the brain. D. If there are language centers on the left side of the brain, any serious stroke affecting that side of the brain

damages at least one of them. E. It is impossible to determine which side of the brain contains a person‘s language centers if the person has not

suffered damage to either side of the brain.

这一题我悟出来了。第二句中的for example不能将前后等同,即第三句中的the stroke不能等同于damage所以,需要有一个前提来说明。D恰好是答案。

而B,如Weiyu所说:―不damage centre-->不impair capability‖恰好补充说明了,―damage centre<-->impair capability‖加上原题,两者是互为充要条件。尽管如此仍不能解决以上D中所说的―举例等同‖问题。

30

GWD-TN-15 3. GWD21-Q3:

Highway Official: When resurfacing our concrete bridges, we should use electrically conductive concrete (ECC) rather than standard concrete. In the winter, ECC can be heated by passing an electric current through it, thereby preventing ice buildup. The cost of the electricity needed is substantially lower than the cost of the deicing salt we currently use. Taxpayer: But construction costs for ECC are much higher than for standard concrete, so your proposal is probably not justifiable on economic grounds.

Which of the following, if true, could best be used to support the highway official‘s proposal in the face of the taxpayer‘s objection?

A. The use of de-icing salt causes corrosion of the reinforcing steel in concrete bridge decks and damage to the

concrete itself, thereby considerably shortening the useful life of concrete bridges. B. Severe icing conditions can cause power outages and slow down the work of emergency crews trying to get power

restored. C. In weather conditions conducive to icing, ice generally forms on the concrete surfaces of bridges well before it

forms on parts of the roadway that go over solid ground. D. Aside from its potential use for de-icing bridges, ECC might also be an effective means of keeping other concrete

structures such as parking garages and airport runways ice free. E. If ECC were to be used for a bridge surface, the electric current would be turned on only at times at which ice was

likely to form. 原因:ECC建设费用比Sc高,

结论:所以这个建议不是建立在经济效用上

削弱:SC会缩短桥的寿命,相比建设费用,因此更加不经济。

4. GWD21-Q4: Which of the following most logically completes the passage?

Appendicitis (inflammation of the appendix) is potentially fatal; consequently, patients with symptoms strongly suggesting appendicitis almost always have their appendix removed. The appropriate surgery is low-risk but performed unnecessarily in about 20 percent of all cases. A newly developed internal scan for appendicitis is highly accurate, producing two

misdiagnoses for every 98 correct diagnoses. Clearly, using this test, doctors can largely avoid unnecessary removals of the appendix without, however, performing any fewer necessary ones than before, since ______.

A. the patients who are correctly diagnosed with this test as not having appendicitis invariably have medical conditions

that are much less serious than appendicitis B. the misdiagnoses produced by this test are always instances of attributing appendicitis to someone who does not, in

fact, have it C. all of the patients who are diagnosed with this test as having appendicitis do, in fact, have appendicitis

D. every patient who is diagnosed with this test as having appendicitis has more than one of the symptoms generally

associated with appendicitis E. the only patients who are misdiagnosed using this test are patients who lack one or more of the symptoms that are

generally associated with appendicitis

文章的要求有两个:

1.largely avoid unnecessary removals of the appendix 2.without performing any fewer necessary ones than before

31

B新方法2%的误诊是将实际未生病的人诊断为生病,换句话说就是实际生病的人一个都不会漏掉,符合without performing any fewer necessary ones than before的要求,同时由于未生病的人诊断为生病的比例只有2%,大大低于原来的20%,因此符合largely avoid unnecessary removals of the appendix C新方法诊断为有病的实际上都有病,这符合largely avoid unnecessary removals of the appendix ,但是新方法诊断为没病的是不是都没病呢,选项没有交代,因此无法判断是否符合without performing any fewer necessary ones than before(如果新方法诊断为没病的实际上有病,这样就会出现漏诊,会performing fewer necessary ones than before,不符合题目要求)

11. GWD7-Q37:(CD-5-37)

Escalating worldwide demand for corn has led to a sharp increase in the market price of corn, and corn prices are likely to remain high. Corn is extensive used as feed for livestock, and because profit margins are tight in the livestock business, many farmers are expected to leave the business. With fewer suppliers, meat prices will surely rise. Nonetheless, observers expect an immediate short-term decrease in meat prices.

Which of the following, if true, most helps to justify the observers‘ expectation?

A. The increase in corn prices is due more to a decline in the supply of corn than to a growth in demand for it. B. Generally, farmers who are squeezed out of the livestock business send their livestock to market much earlier than

they otherwise would. C. Some people who ate meat regularly in the past are converting to diets that include little or no meat. D. As meat prices rise, the number of livestock producers is likely to rise again.

E. Livestock producers who stay in the business will start using feed other than corn more extensively than they did in

the past.

1。D错的原因:producers增加不见得livestock的供给增加而导致价格减低

B对的原因:那些被挤出生意的农民,为结束生意,通常没等livestock长到一定程度就卖掉(因为他们状况一样,被挤出市场时间差不多),从而导致市场短期供应过多而价格短期下降。

lawyer

You think too much. We donnot know whether the price will decrease, let alone it happen at once or after a period of time.

32

14: GWD-18-Q30

The ancient Nubians inhabited an area in which typhus occurs, yet surprisingly few of their skeletons show the usual

evidence of this disease. The skeletons do show deposits of tetracycline, an antibiotic produced by a bacterium common in Nubian soil. This bacterium can flourish on the dried grain used for making two staples of the Nubian diet, beer and bread. Thus, tetracycline in their food probably explains the low incidence of typhus among ancient Nubians. Which of the following is an assumption on which the argument relies?

A. Infectious diseases other than typhus to which the ancient Nubians were exposed are unaffected by tetracycline. B. Tetracycline is not rendered ineffective as an antibiotic by exposure to the processes involved in making bread and

beer. C. Typhus cannot be transmitted by ingesting bread or beer contaminated with the infectious agents of this disease. D. Bread and beer were the only items in the diet of the ancient Nubians which could have contained tetracycline. E. Typhus is generally fatal.

某病在某区常发生,但居民不生病。居民骨头有某抗生素残留,此抗生素可被泥土里的细菌生产。此细菌可在两主要食物中flourish

结论:因此,食物中的抗生素使居民不生病。(细菌->抗生素->食物->不生病)

Gap: 细菌可在食物里flourish,可是抗生素能吗?是抗生素使不生病,不是细菌,所以概念转换的gap存在。 假设:B

------------------------------------------------------------------------------------------------------------ 21. GWD21-Q21:

Which of the following most logically completes the passage?

Mastitis is an infection of the udder in cows that, although not dangerous, causes them to give poor-quality milk. Most cases of mastitis are caused by the bacterium Staphylococcus aureus, against which antibiotics are ineffective. However, a related bacterium, Staphylococcus simulans, quickly clears up even severe infections of S. aureus simply by displacing the bacteria. S. simulans is nevertheless unlikely to become the preferred treatment for cases of mastitis, since ______.

A. certain dangerous bacteria that normally cannot take hold can do so in the presence of S. simulans

B. the current resistance of S. aureus to antibiotics derives in part from a past pattern of overuse of those antibiotics C. the milk from cows infected with S. aureus is generally not of such poor quality as to be entirely unusable D. careful farming practice can reduce the incidence of mastitis to a minimum

E. the only symptom of mild cases of mastitis is the deterioration in the quality of the milk produced 结论:SS不会被用来治疗M病。特殊性:是不用SS,不是其他,是治疗M病,不是干其他事情。 原因:应该涉及SS和M.

------------------------------------------------------------------------------------------------------------ 22. GWD-18-Q38

A survey of entrepreneurs who started companies last year shows that while virtually all did substantial preparatory research and planning, only half used that work to produce a formal business plan. Since, on average, the entrepreneurs without formal plans secured the capital they needed in half the time of those with plans, these survey results indicate that, in general, formal plans did not help the entrepreneurs who produced them to secure the capital they needed. Which of the following, if true, most seriously weakens the argument?

A. Companies started by entrepreneurs who had used formal business plans to attract investment were on the whole

as profitable in their first year as were companies started by entrepreneurs who had not produced such plans. 无关 B. In surveys of entrepreneurs who have attempted without success to raise sufficient capital, more than half of the

respondents indicate that they have produced a formal business plan. 增强 C. Among the entrepreneurs surveyed, those who did not produce formal business plans sought and received a much

larger proportion of their capital from investors with whom they had a long-standing business relationship.

33

D. The entrepreneurs surveyed who did not produce a formal business plan spent nearly as much time doing

preparatory research and planning as the entrepreneurs who produced plans. E. The entrepreneurs who produced business plans generally reported later that the process of writing the plan had

increased their confidence that their company would succeed. 结论:所以,formal plan没有帮助secure the capital.

论据:因为,平均,没有formal plan的企业家使用了更少的时间来获得资金。 削弱:可能是他因导致这些企业家时间少,formal plan的作用无法从论据体现。

------------------------------------------------------------------------------------------------------------

34

28. GWD21-Q28:

Since it has become known that several of a bank‘s top executives have been buying shares in their own bank, the bank‘s depositors, who had been worried by rumors that the bank faced impending financial collapse, have been greatly relieved. They reason that since top executives evidently have faith in the bank’s financial soundness, those worrisome rumors must be false. They might well be overoptimistic, however since corporate executives have sometimes bought shares in their own company in a calculated attempt to dispel negative rumors about the company‘s health. In the argument given, the two boldfaced portions play which of the following roles?

A. The first summarizes the evidence used in the reasoning called into question by the argument; the second states

the counterevidence on which the argument relies. B. The first summarizes the evidence used in the reasoning called into question by the argument; the second is an

intermediate conclusion supported by that evidence. C. The first is an intermediate conclusion that forms part of the reasoning called into question by the argument; the

second is evidence that undermines the support for this intermediate conclusion. D. The first is an intermediate conclusion that forms part of the reasoning called into question by the argument; the

second is the main conclusion of the argument. E. The first is an intermediate conclusion that forms part of the reasoning called into question by the argument; the

second states a further conclusion supported by this intermediate conclusion.

Evidence:several of a bank's top executives have been buying shares in their own bank

-->Intermediate conclusion:top executives evidently have faith in the bank’s financial soundness-->those worrisome rumors must be false

Main conclusion: the investors are wrong,They might well be overoptimistic

since在这里是because的意思,但是后面的句子并不是一个evidence,真正的evidence是it has become known that several of a bank‘s top executives have been buying shares in their own bank。而也是一个推论,由这个推论,引出最后的结论those worrisome rumors must be false.这都是depositors的观点。 D中的main conclusion是作者的最后观点。

------------------------------------------------------------------------------------------------------------ 30. GWD21-Q30:

In the past, most children who went sledding in the winter snow in Verland used wooden sleds with runners and steering bars. Ten years ago, smooth plastic sleds became popular; they go faster than wooden sleds but are harder to steer and slow. The concern that plastic sleds are more dangerous is clearly borne out by the fact that the number of children injured while sledding was much higher last winter than it was ten years ago.

Which of the following, if true in Verland, most seriously undermines the force of the evidence cited?

A. A few children still use traditional wooden sleds.

B. Very few children wear any kind of protective gear, such as helmets, while sledding. C. Plastic sleds can be used in a much wider variety of snow conditions than wooden sleds can. D. Most sledding injuries occur when a sled collides with a tree, a rock, or, another sled.

E. Because the traditional wooden sled can carry more than one rider, an accident involving a wooden sled can result

in several children being injured.

a,一些孩子还在用老式的木头东西。这个跟危险与否没有关系。 b,滑雪时候,没有孩子用防护装置。大家都没有用,对说明没有影响. c,塑料的比木头的要用的广泛。

d,大多数滑雪事故是孩子跟其他物品碰撞时发生的。事故原因,跟危险无关。

35

e,因为木头的能载多人,所以木头的发生事故能造成多人受伤。说的是木头的,没有提到塑料的。

这样看来的话,c要正确,数学问题了,同比,基数大了,数字就多了,所以造成孩子受伤的不是塑料的更危险. 偶一直想不明白为啥D不对,觉得D解释了事故的原因是别的东东造成的,跟塑料危险无关啊,是个他因削弱啊。。。

刚刚忽然灵光一闪,明白鸟,就系因为这些事故的原因对于木头和塑料都是同样存在的,并不是塑料所专有

的,故不能用之来解释塑料的事故多是他因造成!bingo!!

------------------------------------------------------------------------------------------------------------ 33. GWD21-Q33:

One of the limiting factors in human physical performance is the amount of oxygen that is absorbed by the muscles from the bloodstream. Accordingly, entrepreneurs have begun selling at gymnasiums and health clubs bottles of drinking water, labeled ―SuperOXY,‖ that has extra oxygen dissolved in the water. Such water would be useless in improving physical

performance, however, since the only way to get oxygen into the bloodstream so that it can be absorbed by the muscles is through the lungs.

Which of the following, if true, would serve the same function in the argument as the statement in boldface?

A. the water lost in exercising can be replaced with ordinary tap water

B. the amount of oxygen in the blood of people who are exercising is already more than the muscle cells can absorb C. world-class athletes turn in record performances without such water

D. frequent physical exercise increases the body‘s ability to take in and use oxygen E. lack of oxygen is not the only factor limiting human physical performance

------------------------------------------------------------------------------------------------------------

36

38. GWD21-Q38:

Because it was long thought that few people would watch lengthy televised political messages, most televised political advertisements, like commercial advertisements, took the form of short messages. Last year, however, one candidate produced a half-hour-long advertisement. During the half hour the advertisement was aired, a substantial portion of the viewing public tuned into the advertisement. Clearly, then, many more people are interested in watching lengthy televised political messages than was previously thought.

Which of the following is an assumption on which the argument depends?

A. The candidate‘s ratings improved significantly as a result of the half-hour-long political advertisement. B. Political advertisements have become increasingly influential in determining voters‘ decisions at the polls. C. Many people would appreciate the opportunity to become better acquainted with political candidates‘ views on

current political issues. D. Most people who are interested in political issues watch television regularly.

E. Most of the viewers who tuned in to the candidate‘s half-hour-long advertisement last year did not change channels

after the first few minutes. 结论:更多的人对长电视政治消息感兴趣。

原因:当半小时的东东播放时,很多人转到了去那个东东。 假设:记得问假设噢,假设条件取非后题干结论就不成立了。

------------------------------------------------------------------------------------------------------------ 40. GWD21-Q40:

A manufacturer of workstations for computer-aided design seeks to increase sales to its most important corporate customers. Its strategy is to publish very low list prices for workstations in order to generate interest among the buyers for those corporations.

Which of the following, if characteristic of the marketplace, would tend to cause the manufacturer‘s strategy to fail?

A. The proposed list prices would seem low to a typical buyer for the manufacturer‘s most important corporate

customers. B. The capabilities of workstations suitable for given jobs are not significantly different among various manufacturers. C. The manufacturer‘s most important corporate customers employ as buyers persons who are very knowledgeable

about prices for workstations for customer-aided design. D. Customers differ significantly in the percentage of resources they can devote to computer workstations.

E. Buyers for corporations that purchase workstations for computer-aided design receive bonuses for negotiating large

discounts from the list price. 要和list price有关。

------------------------------------------------------------------------------------------------------------

37

GWD-TN-16 2. GWD25-Q2.

In two months, the legal minimum wage in the country of Kirlandia will increase from five Kirlandic dollars(KD5.00) Per hour to KD5.50 per hour. Opponents of this increase have argued that the resulting rise in wages will drive the inflation rate up. In fact its impact on wages will probably be negligible, since only a very small proportion of all Kirfandic workers are currently receiving less than KD5.50 per hour.

Which of the following, if true, most seriously weakens the argument?

A. Most people in Kirlandia who are currently earning the minimum wage have been employed at their current jobs for

less than a year. B. Some firms in Kirlandia have paid workers considerably less than KD5.00 per hour, in violation of Kirlandic

employment regulations.

C. Many businesses hire trainees at or near the minimum wage but must reward trained workers by keeping their pay

levels above the pay level of trainees. D. The greatest growth in Kirlandia‘s economy in recent years has been in those sectors where workers earn wages

that tend to be much higher than the minimum wage. E. The current minimum wage is insufficient for a worker holding only one job to earn enough to support a family,

even when working full time at that job. ----------------------------------------------------------------------------------------- 3. GWD25-Q3.

Mel: The official salary for judges has always been too low to attract the best candidates to the job. The legislature‘s move to raise the salary has done nothing to improve the situation, because it was coupled with a ban on receiving money for lectures and teaching engagements.

Pat: No, the raise in salary really does improve the situation. Since very few judges teach or give lectures, the ban will have little or no negative effect.

Pat‘s response to Mel is inadequate in that it

A. attempts to assess how a certain change will affect potential members of a group by providing evidence about its effect

on the current members. B. mistakenly takes the cause of a certain change to be an effect of that change

C. attempts to argue that a certain change will have a positive effect merely by pointing to the absence of negative effects D. simply denies Mel‘s claim without putting forward any evidence in support of that denial

E. assumes that changes that benefit the most able members of a group necessarily benefit all members of that group. 无定论,我支持B,不过A,C都有支持者,汗。

----------------------------------------------------------------------------------------------------- 11.GWD-19-Q30

Criminologist: Some legislators advocate mandating a sentence of life in prison for anyone who, having twice served

sentences for serious crimes, is subsequently convicted of a third serious crime. These legislators argue that such a policy would reduce crime dramatically, since it would take people with a proven tendency to commit crimes off the streets permanently. What this reasoning overlooks, however, is that people old enough to have served two prison sentences for serious crimes rarely commit more than one subsequent crime. Filling our prisons with such individuals would have exactly the opposite of the desired effect, since it would limit our ability to incarcerate younger criminals, who commit a far greater proportion of serious crimes.

In the argument as a whole, the two boldfaced portions play which of the following roles?

A. The first is a conclusion that the argument as a whole seeks to refute; the second is a claim that has been advanced

in support of that conclusion. B. The first is a conclusion that the argument as a whole seeks to refute; the second is the main conclusion of the

argument. C. The first is the main conclusion of the argument; the second is an objection that has been raised against that

38

conclusion.

D. The first is the main conclusion of the argument; the second is a prediction made on the basis of that conclusion. E. The first is a generalization about the likely effect of a policy under consideration in the argument; the second

points out a group of exceptional cases to which that generalization does not apply. ------------------------------------------------------------------------------------------------------- 12. GWD25-Q12.

In Kantovia, physicians‘ income comes from insurance companies, which require physicians to document their decisions in treating patients and to justify deviations from the companies‘ treatment guidelines. Ten years ago physicians were allowed more discretion. Most physicians believe that the companies‘ requirements now prevent them from spending enough time with patients. Yet the average amount of time a patient spends with a physician during an office visit has actually increased somewhat over the last ten years.

Which of the following, if true, most helps to resolve the apparent discrepancy between physicians‘ perceptions and the change in the actual time spent?

A. Patients are more likely to be in a hurry nowadays and are less willing to wait a long time to see their physician. B. Physicians today typically have a wider range of options in diagnosis and treatment to consider with the patient before

prescribing. C. Physicians are increasingly likely to work in group practices, sharing the responsibility of night and weekend work. D. Most patients would rather trust their physicians than their insurance companies to make decisions about their treatment. E. Since the insurance companies pay physicians a set amount for each office visit, it is to physicians‘ financial advantage

to see as many patients as possible. ------------------------------------------------------------------------------------------------------ 19: GWD-23-Q1

A major health insurance company in Lagolia pays for special procedures prescribed by physicians only if the procedure is first approved as ―medically necessary‖ by a company-appointed review panel. The rule is intended to save the company the money it might otherwise spend on medically unnecessary procedures. The company has recently announced that in order to reduce its costs, it will abandon this rule.

Which of the following, if true, provides the strongest justification for the company‘s decision?

A. B. C.

Patients often register dissatisfaction with physicians who prescribe nothing for their ailments.

Physicians often prescribe special procedures that are helpful but not altogether necessary for the health of the patient.

The review process is expensive and practically always results in approval of the prescribed procedure.

D. The company‘s review process does not interfere with the prerogative of physicians, in cases where more than one

effective procedure is available, to select the one they personally prefer. E.

The number of members of the company-appointed review panel who review a given procedure depends on the cost of the procedure.

------------------------------------------------------------------------------------------ 21. GWD25-Q21

Which of the following most logically completes the editorial below? Editorial in Golbindian Newspaper:

For almost three months, opposition parties have been mounting daily street demonstrations in the capital in an effort to pressure the ruling party into calling an election. Though the demonstrations were well attended at first, attendance has declined steadily in recent weeks. However, the decline in attendance does not indicate that popular support for the opposition‘s demands is dropping, since

A. the opposition‘s demands have not changed during the period when the street demonstrations have been mounted. B. No foreign governments have expressed any support for the opposition‘s demands.

39

C. The state-controlled media have ceased any mention of the demonstrations, leaving many citizens outside the capital

with no way of knowing that demonstrations continue. D. There have not recently been any antigovernment demonstrations in cities other than the capital. E. A recent sharp decrease in unemployment has led to increased popular support for the government.

------------------------------------------------------------------------------------------ 22. GWD25-Q22.

Springfield Fire Commissioner: the vast majority of false fire alarms are prank calls made anonymously from fire alarm boxes on street corners. Since virtually everyone has access to a private telephone, these alarm boxes have outlived their

usefulness. Therefore, we propose to remove the boxes. Removing the boxes will reduce the number of prank calls without hampering people‘s ability to report a fire.

Which of the following, if true, most strongly supports the claim that the proposal, if carried out, will have the announced effect?

A. The fire department traces all alarm calls made from private telephones and records where they came from. B. Maintaining the fire alarm boxes costs Springfield approximately five million dollars annually.

C. A telephone call can provide the fire department with more information about the nature and size of a fire than can an

alarm placed from an alarm box. D. Responding to false alarms significantly reduces the fire department‘s capacity for responding to fires. E. On any given day, a significant percentage of the public telephones in Springfield are out of service.

Removing the boxes will reduce the

number of prank calls without hampering people’s ability to report a fire. that's the point

选C

A是无关选项。理由:题干说the vast majority of false fire alarms are prank calls made anonymously from

fire alarm boxes on street corners.是说假警报绝大部分都是从fire alarm box出来的,注意:人家可没说私人电话有可能报假警哦。所以,蓝色部分就不在我们考虑范围了Removing the boxes will reduce the number of prank calls without hampering people‘s ability to report a fire C说电话比火警箱的好处则可以加强绿色部分

------------------------------------------------------------------------------------------ 30. GWD25-Q30.

Museums that house Renaissance oil paintings typically store them in environments that are carefully kept within narrow margins of temperature and humidity to inhibit any deterioration. Laboratory tests have shown that the kind of oil paint used in these paintings actually adjusts to climatic changes quite well. If, as some museum directors believe, paint is the most sensitive substance in these works, then by relaxing the standards for temperature and humidity control, museums can reduce energy costs without risking damage to these paintings. Museums would be rash to relax those

standards, however, since results of preliminary tests indicate that gesso, a compound routinely used by Renaissance artists to help paint adhere to the canvas, is unable to withstand significant variations in humidity. In the argument above, the two portions in boldface play which of the following roles?

A. The first is an objection that has been raised against the position taken by the argument; the second is the position

taken by the argument. B. The first is the position taken by the argument; the second is the position that the argument calls into question. C. The first is a judgment that has been offered in support of the position that the argument calls into question; the second

40

is a circumstance on which that judgment is, in part based.

D. The first is a judgment that has been offered in support of the position that the argument calls into question; the second

is that position. E. The first is a claim that the argument calls into question; the second is the position taken by the argument. ----------------------------------------------------------------------------------------- 32. GWD-23-Q10

When trying to identify new technologies that promise to transform the marketplace, market researchers survey the

managers of those companies that are developing new technologies. Such managers have an enormous stake in succeeding, so they invariably overstate the potential of their new technologies. Surprisingly, however, market researchers typically do not survey a new technology‘s potential buyers, even though it is the buyers-not the producers-who will ultimately determine a technology‘s commercial success.

Which of the following, if true, best accounts for the typical survey practices among market researchers?

A. If a new technology succeeds, the commercial benefits accrue largely to the producers, not to the buyers, of that

technology. B. People who promote the virtues of a new technology typically fail to consider that the old technology that is

currently in use continues to be improved, often substantially. C. Investors are unlikely to invest substantial amounts of capital in a company whose own managers are skeptical

about the commercial prospects of a new technology they are developing. D. The potential buyers for not-yet-available technologies can seldom be reliably identified.

E. The developers of a new technology are generally no better positioned than its potential buyers to gauge how

rapidly the new technology can be efficiently mass-produced.

Q10 类型 Paradox

题目提出的是一个矛盾: 1. 在衡量新技术的市场潜力时,研发经理的态度是有偏向的.(这个我深有体会..呵呵..)

2. 但是市场通常不调查新技术的最终用户(而是调查研发经理).尽管最终用户的影响决定该技术的前景. Paradox的正确选项要能解释矛盾的双方面,即1,2要同时成立. D:指出新技术的最终用户是很难identify的.1,2同时成立.

D有让1,2都成立吗?好象没有,只使2成立了,不过只要充分即可,所以对。 ----------------------------------------------------------------------------------------- 36. GWD25-Q36

The chemical adenosine is released by brain cells when those cells are active. Adenosine then binds to more and more sites on cells in certain areas of the brain, as the total amount released gradually increases during wakefulness. During sleep, the number of sites to which adenosine is bound decreases. Some researchers have hypothesized that it is the cumulative binding of adenosine to a large number of sites that causes the onset of sleep.

Which of the following, if true, provides the most support for the researchers‘ hypothesis?

A. Even after long periods of sleep when adenosine is at its lowest concentration in the brain, the number of brain cells

bound with adenosine remains very large. B. Caffeine, which has the effect of making people remain wakeful, is known to interfere with the binding of adenosine to

sites on brain cells. C. Besides binding to sites in the brain, adenosine is known to be involved in biochemical reactions throughout the body. D. Some areas of the brain that are relatively inactive nonetheless release some adenosine.

E. Stress resulting from a dangerous situation can preserve wakefulness even when brain levels of bound adenosine are

high.

设A为:adenosine;B为:sleep;

原文结论说的是:有A则有B:A——> B:

adenosine累积到大量的sites时,启动了睡眠;

41

选项B

说:

没有A就没有B

:无

A——>无 B:

Caffeine,妨碍了adenosine累积(无A),因此人保持清醒(不想睡,既无B),从反面支持了原来的结论。

---------------------------------------------------------------------------------------- 40. GWD-23-Q14

Plantings of cotton bioengineered to produce its own insecticide against bollworms, a major cause of crop failure, sustained little bollworm damage until this year. This year the plantings are being seriously damaged by bollworms. Bollworms,

however, are not necessarily developing resistance to the cotton‘s insecticide. Bollworms breed on corn, and last year more corn than usual was planted throughout cotton-growing regions. So it is likely that the cotton is simply being overwhelmed by corn-bred bollworms.

In evaluating the argument, which of the following would be most useful to establish?

A. Whether corn could be bioengineered to produce the insecticide

B. Whether plantings of cotton that does not produce the insecticide are suffering unusually extensive damage from

bollworms this year C. Whether other crops that have been bioengineered to produce their own insecticide successfully resist the pests

against which the insecticide was to protect them D. Whether plantings of bioengineered cotton are frequently damaged by insect pests other than bollworms

E. Whether there are insecticides that can be used against bollworms that have developed resistance to the insecticide

produced by the bioengineered cotton

这题说, 棉花有抵制那个虫子的anti

但是还是受害了,因为玉米是虫子变得很多 这样的话,也就是说有两个结论

玉米是棉花受害,棉花本身的anti有用

B说的是,如果棉花没有anti的话,虫子会不会对他造成极大伤害

这样,如果是, 说明anti有用, 如果不是,说明anti没用 于是, 按照原文的意思就是, 如果是,原文成立, 如果不是,原文不成立

这个题的中心是棉花的anti是自己没用呢?还是被corn给盖过了

所以要evaluate他,就要看那些没有anti的棉花是不是遭受了更大的伤害

My opinion: 此题是否偷换了概念。Plantings of cotton bioengineered to produce its own insecticide against bollworms与This year the plantings是否是同一个东西。

----------------------------------------------------------------------------------

42

GWD-TN-17

4. GWD26-Q4

Consumers planning to buy recreational equipment tend to buy higher quality, more expensive equipment when the

economy is strong than when it is weak. Hill and Dale is a business that sells high-quality, expensive camping and hiking equipment in Boravia. Although all the signs are that Boravia‘s economy is now entering a period of sustained strength, the managers of the business do not expect a substantial increase in sales.

Which of the following, if true, would provide the strongest justification for the managers‘ judgment?

A. A significant proportion of Hill and Dale‘s sales are made to customers who enter the store in order to buy one particular

item but, once there, find other items to buy as well. B. In Boravia when the economy is strong, those who might otherwise go camping tend to take vacations overseas. C. The economic upturn is likely to allow Boravia‘s national parks, where most of the camping and hiking is done, to

receive extra funding to improve their visitor facilities. D. Advances in materials technology have led to the development of hiking and camping equipment that is more

comfortable and lightweight than before. E. Many people in Boravia not only are committed to preserving the country‘s wilderness areas but also are interested in

spending some time in them. ------------------------------------------------------------------------------------------------------- 10. GWD26-Q10

Lawmaker: Raising taxes is not the only means of reducing government debt. The government‘s stockpile of helium is worth 25 percent more, at current market prices, than the debt accumulated in acquiring and storing it. Therefore, by selling the helium, the government can not only pay off that debt but reduce its overall debt as well. Which of the following is an assumption on which the argument depends? A. The government has no current need for helium.

B. Twenty-five percent of the debt the government has accumulated in stockpiling helium is not an insignificant portion of

the government‘s total debt. C. It is not in the lawmaker‘s interest to advocate raising taxes as a means of reducing government debt. D. Attempts to sell the government‘s helium will not depress the market price of helium by more than 25 percent. E. The government will not incur any costs in closing its facilities for stockpiling helium.

有两个方向的想法。首先A去非。美国政府有hel的需要,这并不意味着美国就不能卖hel,题目的主线是卖hel 可不可以还债。即使美国政府需要hel,他还可以去卖hel还债。 除非说All of the hel available is to be needed by US industry.这样hel就不能卖了。 C得说法是卖了25%得不会导致价钱降低。取非以后以后就是说卖了25%会导致hel无利可图。题目的主线是卖hel有利可图,所以卖hel还债。 C非,就是断桥削弱。

3.26-10的BY出现在结论,是作者得出结论的前提,即作者在结论成立的前提是SELLING成功.所以不能SELLING的情况和作者的结论无关.

4.D的取非是:Attempts to sell the government’s helium will depress the market Price of helium by mor

e than 25 percent. 即贬值超过25%.原来比债务高于25%.这样贬值后就不能全部还债(pay off)

5.考试时时间很短,所以不可能很细计算,意味着出题人不会考细的东西,只会考大方向.所以不可能答案错在小计算上 lawyer

43

--------------------------------------------------------------------------------------------------- 18. GWD26-Q18

A medieval manuscript called L contains all eighteen extant tragedies by the Greek playwright Euripides. Of these, ten called the ―select plays,‖ are accompanied in L by ancient commentaries and also appear in other medieval manuscripts; this group includes some of Euripides‘ best-known works, including the Medea. The other eight, which appear in alphabetical order, without commentary. The Electra is one of the alphabeticals.

Which of the following can be reliably concluded on the basis of the Statements given?

A. Only Euripides‘ best-known works are accompanied by ancient commentaries in extant medieval manuscripts. B. The select plays are accompanied by ancient commentaries because they were the best known of Euripides‘ works. C. No commentaries were written about Euripides‘ Electra in ancient times.

D. Euripides‘ Medea never appears in medieval manuscripts unaccompanied by ancient commentary. E. Euripides‘ Electra does not appear accompanied by a commentary in any extant medieval manuscript.

想啊想,终于想明白为什E对。

ABCD都可以排除,对于E,有的G友说应该把any 改成all才对。我想E这个句子存在一个理解问题:

Euripides’ Electra does not appear accompanied by a comentary in any extant medieval manuscript.

若理解成:在任何一部extant medieval manuscript中E都不伴随评论出现。则这道题没答案了。

但我想GMAC愿意是: 强调does not appear accompanied by a comentary in any extant medieval manuscript

不是在任何一部extant medieval manuscript中,E都是以伴随评论出现的。因此正符合原文意思。

没选E的同学可能很难一下子把原来的逻辑转回来,唯一的办法就是多念几遍这个句子了。

******************************************************* 39楼的mm,你好牛. 我觉得你的解释是正确的!!!也是唯一合理的!

其实E想说: Euripides的Electra这个剧本,并不每次都是伴随这评论出现在当今现存的手稿里的( 起码在L这部手稿里,就没带评论).

------------------------------------------------------------------------------------------------------- 19. GWD-23-Q32

Excavations of the Roman city of Sepphoris have uncovered numerous detailed mosaics depicting several readily identifiable animal species: a hare, a partridge, and various Mediterranean fish. Oddly, most of the species represented did not live in the Sepphoris region when these mosaics were created. Since identical motifs appear in mosaics found in other Roman cities, however, the mosaics of Sepphoris were very likely created by traveling artisans from some other part of the Roman Empire.

Which of the following is an assumption on which the argument depends?

A. The Sepphoris mosaics are not composed exclusively of types of stones found naturally in the Sepphoris area. B. There is no single region to which all the species depicted in the Sepphoris mosaics are native.

C. No motifs appear in the Sepphoris mosaics that do not also appear in the mosaics of some other Roman city. D. All of the animal figures in the Sepphoris mosaics are readily identifiable as representations of known species. E. There was not a common repertory of mosaic designs with which artisans who lived in various parts of the Roman

Empire were familiar.

44

I think the answer should be E.

In the arg ,it is said that the animal which were depicted in mosaic had never been live when the mosaic was created.So author think it was created by someone came from another place.

If there is a place where most of artists in different areas could saw and share the information.or there was a common design everybody was familar.Then the artists who live in S could make the motifs ,too.Then the conclusion would be weaken.

应该用取非削弱吧 结论是mosaics是从别的地方带来的 如果E取非就是不同地方都有相同的设

计 也就削弱了结论 应该是选E

我的意见:assumption题还是要先看结论,最后一句是结论。看前半部分,Since identical motifs appear

in mosaics found in other Roman cities,有这个原因作者推出结论。E取非就是提供信息,让从这个推不出结论。 ------------------------------------------------------------------------------------------------------- 22. GWD26-Q22

People who have spent a lot of time in contact with animals often develop animal-induced allergies, some of them quite serious. In a survey of current employees in major zoos, about 30 percent had animal-induced allergies. Based on this

sample, experts conclude that among members of the general population who have spent a similarly large amount of time in close contact with animals, the percentage with animal-induced allergies is not 30 percent but substantially more. Which of the following, if true, provides the strongest grounds for the experts‘ conclusion?

A. A zoo employee who develops a serious animal-induced allergy is very likely to switch to some other occupation. B. A zoo employee is more likely than a person in the general population to keep one or more animal pets at home C. The percentage of the general population whose level of exposure to animals matches that of a zoo employee is quite

small. D. Exposure to domestic pets is, on the whole, less likely to cause animal-induced allergy than exposure to many of the

animals kept in zoos. E. Zoo employees seldom wear protective gear when they handle animals in their care.

嗯,其实之前我也有想过lucy说的这个问题,但系再又觉得动物管理员不会有多少吧,而普通人群中与动物有亲密接触的人这个基数其实是很大的,即使加入了几个动物管理员对比例应该不会产生什么影响吧。。。

偶认为这个underestimate是相对的,并不说这个survey是不准的意思,只是就目前的情况而论,的确存在一些有过敏症的动物管理员已经离职而没有算到survey里面,所以才会认为这个比例应该更高一点才贴近普通人群的实际比例。

同意,动物园的30%其实低估了,因为动物园员工由于一些人受不了过敏需要换工作,大量新员工需要补充,这使得动物园的过敏员工比例可以维持在30%的低水平

------------------------------------------------------------------------------------------------ GWD27-Q28 (GWD-26-Q22)

People who have spent a lot of time in contact with animals often develop animal-induced allergies, a significant percentage of which are quite serious. In a survey of current employees in major zoos, about 30 percent had animal-induced allergies. However, a zoo employee who develops a serious animal-induced allergy is very likely to switch to some other occupation. Which of the following hypotheses receives the strongest support from the information given?

A. The incidence of serious animal-induced allergies among current zoo employees is lower than that among the

general population. B. Zoo employees tend to develop animal-induced allergies that are more serious than those of other people who

spend equally large amounts of time with animals. C. Exposure to domestic pets is, on the whole, less likely to cause animal-induced allergy than is exposure to the kinds

45

of animals that are kept in zoos.

D. There is no occupation for which the risk of developing an animal-induced allergy is higher than 30 percent. E. Among members of the general population who have spent as much time with animals as zoo employees typically

have, the percentage with animal-induced allergies is significantly more than 30 percent. ------------------------------------------------------------------------------------------------------- 29. GWD26-Q29 Educational Theorist:

Recent editorials have called for limits on the amount of homework assigned to children. They point out that free-time

activities play an important role in childhood development and that large amounts of homework reduce children‘s free time, hindering their development. But the average homework time for a ten year old, for example, is little more than 30 minutes per night. Clearly, therefore, there is no need to impose the limits these editorials are calling for. Which of the following is an assumption on which the educational theorist‘s argument relies?

A. The free-time activities that ten year olds engage in most are all approximately equally effective at fostering

development B. Regularly doing homework assignments improves children‘s academic performance.

C. Individual teachers are not the best judges of how much homework to assign the children they teach

D. In most schools, if not all, the homework assignments given are of a length that does not diverge widely from the

average. E. Free-time activities rarely teach children skills or information that they can use in their academic work.

My opinion: 从此题好象可以看出,对于法规/limit/,用average来argue是不行的,因为限制并不是对average起作用的,而是对最低或者最高起作用的。??

-------------------------------------------------------------------------------------------------------

31. GWD26-Q31 此题要小心,不能一下子排除A,有时候靠感觉是不对的,而且关键要有理才行。 Which of the following most logically completes the argument?

When people engage in activities that help others, their brain releases endorphins, the brain‘s natural opiates, which induce in people a feeling of well-being. It has been suggested that regular release of endorphins increases people‘s longevity. And a statistic on adults who regularly engage in volunteer work helping others shows that they live longer, on average, than adults who do not volunteer. However, that statistic would be what we would expect even if volunteering does not boost longevity, because .

A. in the communities studied, women were much more likely to do regular volunteer work than men were, and women

tend to live longer than men do. B. the number of young adults who do regular volunteer work is on the increase

C. the feelings of well-being induced by endorphins can, at least for a time, mask the symptoms of various conditions and

diseases, provided the symptoms are mild. D. it is rare for a person to keep up a regular schedule of volunteer work throughout his or her life.

E. Some people find that keeping a commitment to do regular volunteer work becomes a source of stress in their lives.

支持A

理解原文最后一句话最关键:即使volunteering不能增加人的寿命,那组统计数据也一样,没有变化,都是volunterring比没有voluntering的人长寿.说明这组人比较特殊.这些人寿命本来就长,不作voluntering寿命也长. A的意思是那组数据统计的是妇女,而他们通常比较长寿.所以拿这些人去统计,结果会是比平均成年人长命,即使扣除他们做的volunteering使他们长寿的因素. lawyer

------------------------------------------------------------------------------------------------------- 39. GWD3-Q38:(CD2-38)

46

Kate: The recent decline in numbers of the Tennessee warbler, a North American songbird that migrates each fall to coffee plantations in South America, is due to the elimination of the dense tree cover that formerly was a feature of most South American coffee plantations.

Scott: The population of the spruce budworm, the warbler‘s favorite prey in North America, has been dropping. This is a more likely explanation of the warbler‘s decline.

Which of the following, if true, most seriously calls Scott‘s hypothesis into question?

A. The numbers of the Baltimore oriole, a songbird that dose not eat budworms but is as dependent on South

American coffee plantations as is the Tennessee warbler, are declining. B. The spruce-budworm population has dropped because of a disease that can infect budworms but not Tennessee

warblers. C. The drop in the population of the spruce budworm is expected to be only temporary.

D. Many Tennessee warbler have begun migrating in the fall to places other than traditional coffee plantations. E. Although many North American songbirds have declined in numbers, no other species has experienced as great a

decline as has the Tennessee warbler. ------------------------------------------------------------------------------------------------------- 41.: GWD-23-Q39

In Gandania, where the government has a monopoly on tobacco sales, the incidence of smoking-related health problems has risen steadily for the last twenty years. The health secretary recently proposed a series of laws aimed at curtailing tobacco use in Gandania. Profits from tobacco sales, however, account for ten percent of Gandania‘s annual revenues. Therefore, Gandania cannot afford to institute the proposed laws.

Which of the following, if true, most seriously weakens the argument? A. All health care in Gandania is government-funded.

B. Implementing the proposed laws is not likely to cause a significant increase in the amount of tobacco Gandania exports. C. The percentage of revenue Gandania receives from tobacco sales has remained steady in recent years. D. Profits from tobacco sales far surpass any other single source of revenue for the Gandanian government. E. No government official in Gandania has ever previously proposed laws aimed at curtailing tobacco use.

找了个牛人的解释:烟草赚钱是事实,不能ban是结论 说烟草导致病增多

然后因为烟草是收入, 所以不能ban烟草

这里的主要推力是,因为烟草带来钱,所以不能ban weaken要去反对说烟草可以ban

C,说烟草收入稳定。。 这样要是ban了,不就木有钱拉 所以C是support A是这么说的,政府要出钱看病

这样不ban烟草,导致大家得病,于是政府就用卖烟的钱来给大家治病 那不是还不如不卖烟阿

-------------------------------------------------------------------------------------------------

47

GWD-TN-18

2. GWD-27-Q2:

Electronic computer chips made of tiny silicon wafers now regularly contain millions of electronic switches. Unfortunately, electronic switches that are this small cannot withstand intense radiation. Micro-Mechanics plans to produce a chip that, because it uses only microscopic mechanical switches, will be invulnerable to radiation damage. The switches will, however, be slower than electronic switches and the chip will contain only 12,000 switches.

For there to be a market for Micro-Mechanics‘ chip as a result of the apparent advantage described above, each of the following would have to be true EXCEPT:

A. There will be applications in which the speed attainable by an electronic switch is not essential.

B. Switches used on electronic chips that contain only 12,000 switches are more vulnerable to radiation damage than

the switches on Micro-Mechanics‘ chip will be. C. There will be applications for computer chips in environments where the chips may have to survive intense radiation. D. Some devices in which computer chips will be used will have other components that will be able to function during

or after exposure to radiation. E. Manufacturers are able to protect electronic computer chips against exposure to intense radiation, where this

protection is necessary. 解析:原文推理:electronic switches不能忍受intense radiation,Micro-Mechanics‘ chip可以忍受,但是slower than electronic switches,结论说Micro-Mechanics‘ chip有市场,也就是说就算慢也可以

思路:要加强有市场,就要强调Micro-Mechanics‘ chip能够抗击radiation damage的优点,淡化速读慢的缺点。 A:说速度不重要,自然就去除了慢这个缺点,也就有市场了。

B:相同的速读下electronic chips比Micro-Mechanics‘ chip更脆弱,加强了Micro-Mechanics‘ chip抗击radiation damage的能力

C:chips may have to survive intense radiation说明抗击radiation damage很重要,因此有市场

D:错误是认为其他组成部分是否能够忍受radiation damage和Micro-Mechanics‘ chip有市场无关。但是这个选项说明使用Micro-Mechanics‘ chip没有副作用。之前使用electronic switches的时候因为不能忍受intense radiation,所以只能在弱radiation的情况下使用,现在换成了Micro-Mechanics‘ chip,也就是说可以在强radiation下使用,但是如果装置中的其他零件不能忍受强radiation的话,就算是换上了Micro-Mechanics‘ chip也不能用,所以也就没有市场了。这个选项排除了这种情况。 E:正确,很明显的错误,electronic computer chips可以被保护不受radiation damage,那么就不需要Micro-Mechanics‘ chip了,削弱结论

--------------------------------------------------------------------------------------------------- 8: GWD-23-Q40

Political advocacy groups have begun to use information services to disseminate information that is then accessed by the public via personal computer. Since many groups are thus able to bypass traditional news sources, whose reporting is selective, and to present their political views directly to the public, information services present a more balanced picture of the complexities of political issues than any traditional news source presents. Which of the following is an assumption on which the argument above depends?

A. Information services are accessible to enough people to ensure that political advocacy groups can use these

services to reach as large a percentage of the public as they could through traditional news sources. B. People could get a thorough understanding of a particular political issue by sorting through information provided by

several traditional news sources, each with differing editorial biases. C. Information on political issues disseminated through information services does not come almost entirely from

advocacy groups that share a single bias. D. Traditional news sources seldom report the views of political advocacy groups accurately.

E. Most people who get information on political issues from newspapers and other traditional news sources can readily

identify the editorial biases of those sources.

48

------------------------------------------------------------------------------------------------------- 9. GWD27-Q9:

In order to raise revenue, the federal government planned a tax amnesty program that allows tax delinquents to pay all owed tax without added financial penalty. However, economists projected that the federal government would collect a far lower percentage of total tax owed by delinquents than did state governments implementing similar programs. Which of the following, if true, would most contribute to an explanation of the economists‘ projections?

A. Tax amnesty programs are only successful if they are widely publicized.

B. Most people who honestly pay their state tax are equally honest in paying their federal tax.

C. Although federal tax delinquents usually must pay high financial penalties, the states require far lower financial

penalties. D. The state tax rate varies considerably from state to state, but the federal tax is levied according to laws which apply

to citizens of all the states. E. Unlike most federal tax delinquents, most state tax delinquents fail to pay state tax because of an oversight rather

than a decision not to pay. ------------------------------------------------------------------------------------------------------- 16: GWD-29-Q8

Which of the following, if true, provides evidence that most logically completes the argument below?

According to a widely held economic hypothesis, imposing strict environmental regulations reduces economic growth. This hypothesis is undermined by the fact that the states with the strictest environmental regulations also have the highest economic growth. This fact does not show that environmental regulations promote growth, however, since ______.

A. those states with the strictest environmental regulations invest the most in education and job training

B. even those states that have only moderately strict environmental regulations have higher growth than those with

the least-strict regulations C. many states that are experiencing reduced economic growth are considering weakening their environmental

regulations D. after introducing stricter environmental regulations, many states experienced increased economic growth E. even those states with very weak environmental regulations have experienced at least some growth ------------------------------------------------------------------------------------------------------- 20. GWD27-Q20:

Critics of certain pollution-control regulations have claimed that the money spent over the last decade in order to reduce emissions of carbon monoxide and of volatile organic compounds has been wasted. The evidence they offer in support of this claim might appear compelling: despite the money spent, annual emissions of these pollutants have been increasing steadily. This evidence is far from adequate, however, since over the last decade a substantial number of new industrial facilities that emit these pollutants have been built. In the reasoning given, the two portions in boldface play which of the following roles?

A. The first identifies a claim that the reasoning seeks to show is false; the second is evidence that has been cited by

others in support of that claim. B. The first identifies a claim that the reasoning seeks to show is false; the second is a position for which the reasoning

seeks to provide support. C. The first is a position that the reasoning contends is inadequately supported by the evidence; the second is a

position for which the reasoning seeks to provide support. D. The first is a position that the reasoning contends is inadequately supported by the evidence; the second is evidence

used to support the reasoning‘s contention. E. The first is a position that the reasoning contends is inadequately supported by the evidence; the second is evidence

that has been used to support that position. ------------------------------------------------------------------------------------------------------- 22. GWD27-Q22:

49

Whales originated in the freshwater lakes and rivers of ancient Asia about sixty million years ago. Not until about ten million years later did species of whales develop specialized kidneys enabling them to drink salt water. Although fossil evidence shows that some early whale species that lacked such kidneys sometimes swam in the Earth‘s saltwater oceans, these species must have had to return frequently to freshwater rivers to drink. Which of the following is most strongly supported by the information given?

A. Fossils of whale species dating from between sixty million and fifty million years ago will not be found on continents

that were at the time separated from ancient Asia by wide expanses of ocean. B. Among whale fossils that date from later than about fifty million years ago, none are fossils of whale species that

drank only fresh water. C. Fossils of whale species that drank fresh water will not be found in close proximity to fossils of whale species that

drank salt water. D. The earliest whales that drank salt water differed from fresh-water-drinking whales only in their possession of

specialized kidneys. E. Between sixty million and fifty million years ago, the freshwater lakes and rivers in which whales originated were

gradually invaded by salt water.

原文没有说明5000万年后的鲸鱼是否只喝淡水,只说到可以喝海水;B属于无关选项。 虽然这题A Fossils of whale species dating from between sixty million and fifty million years ago will not be found on continents that were at the time separated from ancient Asia by wide expanses of ocean.

6000万年前到5000万年前的鲸鱼化石不会在那个时期被宽阔海洋从古亚洲分离出去的大陆上发现。

大家的理解是鲸鱼不可能游过海洋,因为没法过滤海水中的盐分;但是,其实原文就有说是从湖泊和河流进化来的,那么分离出去的大陆也是有可能有湖泊和河流,也有可能恰好这些湖泊和河流都有古鲸鱼,那么就有发现化石的可能;所以A也有些不充分,但却是最好的选项。

只喝淡水的鲸鱼可以到海里,但是要回去淡水里喝水,所以就不可以在海里呆太久。 因为鲸鱼是在asia起源的,所以在他们能靠海水生活之前,他们不可能出现在其他和asia被海隔开的大州上,游不过去嘛,会干死的。A中就给出了这个信息。

------------------------------------------------------------------------------------------------ 28: GWD-29-Q9

Manatees, aquatic mammals inhabiting Florida‘s rivers and coastal waters, swim close to the surface and are frequently killed in collisions with boats. To address the problem, boat traffic in manatee-populated waters is being required to maintain very low speeds. Unfortunately, manatees are unable to hear low-pitched sounds and a boat‘s sound lowers in pitch as the boat slows. Therefore, this approach may in fact make things worse rather than better. Which of the following, if true, casts most doubt on the conclusion?

A. The areas where boats would have to maintain low speeds were decided partly on the basis of manatee-population

estimates and partly from numbers of reported collisions between manatees and boats. B. Because the water hyacinth that manatees feed on grows best in water that is nearly still, water hyacinth beds can

be disturbed or damaged by fast-moving boat traffic. C. Over the last several decades, boat traffic in Florida‘s coastal waters has been increasing almost continuously and

now represents the greatest threat to the endangered manatee population. D. The sound of a boat engine generally travels much further under water than it does through the air.

50

本文来源:https://www.bwwdw.com/article/9kwx.html

Top